Search results

  • * [https://pimathcontest.com/ Pi Math Contest] is a math competition for elementary, middle, and high school *[[RSM Foundation International Math Contest]] is a two-round Olympiad-style contest for students in grades 3-8. ([https://rsmfound
    4 KB (473 words) - 16:14, 1 May 2024
  • * [https://pimathcontest.org/ Pi Math Contest] is a two-round math competition for elementary and middle school students. *[[RSM Foundation International Math Contest]] is a two-round Olympiad-style contest for students in grades 3-8 ([https://rsmfounda
    7 KB (792 words) - 10:14, 23 April 2024
  • The [[Triangle Inequality]] says that the sum of the lengths of any two sides of a non[[degenerate]] triangle is greater than the length of the thi ...area]] <math>A</math> and [[perimeter]] <math>P</math>, then <math>\frac{4\pi A}{P^2} \le 1</math>. This means that given a perimeter <math>P</math> for
    7 KB (1,296 words) - 14:22, 22 October 2023
  • An elliptic curve is the set of points <math>(x,y)</math> satisfying some two variable third degree equation. Using certain affine transformations it can ...math> or when this line is tangent to the curve, and hence cuts it in only two points.) We define <math>P+Q</math> to be the reflection of <math>R</math>
    5 KB (849 words) - 16:14, 18 May 2021
  • ...any value. Some examples of real numbers are:<math>1, 2, -23.25, 0, \frac{\pi}{\phi}</math>, and so on. Numbers that are not real are <math>\ 3i</math>, ...n the [[algebraic number]]s and [[transcendental number]]s, although these two classes are best understood as subsets of the [[complex number]]s.
    3 KB (496 words) - 23:22, 5 January 2022
  • ...^2</math>. Both formulas involve the mathematical constant [[pi]] (<math>\pi</math>). We shall explore two of the Greek [[mathematician]] [[Archimedes]] demonstrations of the area of
    9 KB (1,555 words) - 20:05, 2 November 2023
  • ...ath> to two fixed [[focus|foci]] is a constant. (The equivalence of these two definitions is a non-trivial fact.) ...endicular]] to the axis of the the cone, or (in the second definition) the two foci of the ellipse coincide.
    5 KB (892 words) - 21:52, 1 May 2021
  • ...mber]], as proved by Lindemann in 1882) denoted by the Greek letter <math>\pi </math>. ...^2}=\frac{\pi^2}{6}</math>. Some common [[fraction]]al approximations for pi are <math>\frac{22}{7} \approx 3.14285</math> and <math>\frac{355}{113} \ap
    8 KB (1,469 words) - 21:11, 16 September 2022
  • ..., we have <math>\ln (-1)=i\pi</math>. Additionally, <math>\ln (-n)=\ln n+i\pi</math> for positive real <math>n</math>. Two-minute Intro to Logarithms [http://www.youtube.com/watch?v=ey7ttABX9SM]
    4 KB (680 words) - 12:54, 16 October 2023
  • ...n [[decimal notation]], never terminates nor repeats. Examples are <math>\pi, \sqrt{2}, e, \sqrt{32134},</math> etc. There are two types of irrational numbers: algebraic and transcendental.
    3 KB (368 words) - 19:26, 6 June 2015
  • ...as a positive real number). This leaves us with <math>e^{ni\theta} = e^{2\pi ik}</math>. ...math> ni\theta = 2\pi ik</math>. Solving this gives <math> \theta=\frac{2\pi k}n </math>. Additionally, we note that for each of <math> k=0,1,2,\ldots,
    3 KB (558 words) - 21:36, 11 December 2011
  • <cmath> f^{(n)}(z_0)=\frac{n!}{2\pi i} \oint_\Gamma \frac{f(z)\; dz}{(z-z_0)^{n+1}}. </cmath> ...lle's Theorem. If <math>f</math> is an entire function so that there exist two complex numbers <math>a</math> and <math>b</math> such that for every compl
    2 KB (271 words) - 22:06, 12 April 2022
  • |<math>\pi</math>||\pi||<math>\varpi</math>||\varpi||<math>\rho</math>||\rho||<math>\varrho</math> | <math>\Xi</math>||\Xi||<math>\Pi</math>||\Pi||<math>\Sigma</math>||\Sigma||<math>\Upsilon</math>||\Upsilon
    16 KB (2,324 words) - 16:50, 19 February 2024
  • ...can <math>|\pi - |e - | e - \pi|||</math> be expressed in terms of <math>\pi</math> and <math>e?</math> ...}2\pi-2e\qquad\textbf{(C) }2e\qquad\textbf{(D) }2\pi \qquad\textbf{(E) }2\pi +e</math>
    12 KB (1,784 words) - 16:49, 1 April 2021
  • ...ball game was played between two teams, the Cougars and the Panthers. The two teams scored a total of 34 points, and the Cougars won by a margin of 14 po ...they get into their family car, two people sit in the front, and the other two sit in the back. Either Mr. Lopez or Mrs. Lopez must sit in the driver's s
    13 KB (2,058 words) - 12:36, 4 July 2023
  • ...math> is cut into two congruent hexagons, as shown, in such a way that the two hexagons can be repositioned without overlap to form a square. What is <mat How many sets of two or more consecutive positive integers have a sum of <math>15</math>?
    15 KB (2,223 words) - 13:43, 28 December 2020
  • ...0</math>, or <math>1</math>. How many distinct lines pass through at least two members of <math>S</math>? ...rt3}{2} \qquad \text {(D) } \frac {10\pi}{3} - \sqrt3 \qquad \text {(E) }4\pi - 2\sqrt3</math>
    13 KB (1,953 words) - 00:31, 26 January 2023
  • The sum of the two 5-digit numbers <math>AMC10</math> and <math>AMC12</math> is <math>123422</ ...}{4}+\frac{1}{24}\pi\qquad \mathrm{(E) \ } \frac{\sqrt{3}}{4}+\frac{1}{12}\pi </math>
    13 KB (1,955 words) - 21:06, 19 August 2023
  • ...\pi \qquad \text{(C)}\ 2\pi \qquad \text{(D)}\ 3\pi \qquad \text{(E)}\ 3.5\pi</math> Two different positive numbers <math>a</math> and <math>b</math> each differ fr
    12 KB (1,792 words) - 13:06, 19 February 2020
  • The sum of two numbers is <math>S</math>. Suppose <math>3</math> is added to each number a each of the resulting numbers is doubled. What is the sum of the final two
    13 KB (1,957 words) - 12:53, 24 January 2024
  • ...more than a pink pill, and Al's pills cost a total of 546 dollars for the two weeks. How much does one green pill cost? ...) = 8</math> and <math>\clubsuit(123) = 1 + 2 + 3 = 6.</math> For how many two-digit values of <math>x</math> is <math>\clubsuit(\clubsuit(x)) = 3?</math>
    13 KB (1,987 words) - 18:53, 10 December 2022
  • ...{C}) 75+100\pi \quad (\mathrm {D}) 100+100\pi \quad (\mathrm {E}) 100+125\pi</math> ...a display of cans in which the top row has one can and each lower row has two more cans than the row above it. If the display contains 100 cans, how many
    13 KB (2,049 words) - 13:03, 19 February 2020
  • ...e for <math>2</math> dollars. They sell all the candy bars at the price of two for <math>1</math> dollar. What was their profit, in dollars? \mathrm{(A)}\ 80-20\pi \qquad
    12 KB (1,781 words) - 12:38, 14 July 2022
  • ...math>\frac{14^2}{2}-\frac{4^2}{2}-(\frac{5\sqrt{2}}{2})^2\pi=98-8-\frac{25\pi}{2}</math>, which is approximately <math>51</math>. The answer is <math>\bo
    2 KB (262 words) - 21:20, 21 December 2020
  • ...i\theta_n}</math>, where <math>\theta_n=2^n(\theta_0+\frac{\pi}{2})-\frac{\pi}{2}</math>. ...\frac{iz_k}{\overline {z_k}}=i e^{i\theta_k} e^{i\theta_k}=e^{i(2\theta_k+\pi/2)}
    4 KB (660 words) - 17:40, 24 January 2021
  • ...math> is cut into two congruent hexagons, as shown, in such a way that the two hexagons can be repositioned without overlap to form a square. What is <ma How many sets of two or more consecutive positive integers have a sum of 15?
    13 KB (2,028 words) - 16:32, 22 March 2022
  • ...e have that <math>\left(\sin\left(\frac\pi2 - u\right) + i \cos\left(\frac\pi 2 - u\right)\right)^n = \sin n \left(\frac\pi2 -u \right) + i\cos n \left(\ ...left(\cos u + i \sin u\right)^n = \cos nu + i\sin nu</math>. We know that two [[complex number]]s are equal if and only if both their [[real part]] and [
    6 KB (1,154 words) - 03:30, 11 January 2024
  • Setting these two expressions for <math>(OF)^2</math> equal and solving for <math>x</math> (i ...equation yields <math>a^2+b^2 = 290000</math>. We now finish by adding the two original stewart equations and obtaining: <cmath>2(450\sqrt{2})^2 = (500-x)
    13 KB (2,080 words) - 21:20, 11 December 2022
  • ...ruent]] [[circle]]s form a ring with each circle [[externally tangent]] to two circles adjacent to it. All circles are [[internally tangent]] to a circle ...ath>. <math>K = 30^2\pi - 6(10^2\pi) = 300\pi</math>, so <math>\lfloor 300\pi \rfloor = \boxed{942}</math>.
    1 KB (213 words) - 13:17, 22 July 2017
  • ...ation are <math>x = 1 + i^n r</math> for <math>n = 0, 1, 2, 3</math>. The two non-real members of this set are <math>1 + ir</math> and <math>1 - ir</math ...a quadratic, then <math>((x-1)^2+\sqrt{2006})</math> is bound to hold the two non-real roots since the other definitely crosses the x-axis twice since i
    4 KB (686 words) - 01:55, 5 December 2022
  • ...ed. A [[plane]] [[parallel]] to the base of the cone divides the cone into two solids, a smaller cone-shaped solid <math> C </math> and a [[frustum]]-shap ...the [[Pythagorean Theorem]], we get <math>\ell = 5</math> and <math>A = 24\pi</math>.
    5 KB (839 words) - 22:12, 16 December 2015
  • ...sides of the square determined by the vertex <math>(0,0)</math>. Let the two endpoints of the segment have coordinates <math>(x,0)</math> and <math>(0,y ...ed by all of the midpoints is <math>4-4\cdot \left(\frac{\pi}{4}\right)=4-\pi \approx .86</math> to the nearest hundredth. Thus <math>100\cdot k=\boxed{8
    3 KB (532 words) - 09:22, 11 July 2023
  • ...of which are quadrilaterals. A space diagonal is a line segment connecting two non-adjacent vertices that do not belong to the same face. How many space d ...ratio was <math>\frac{300}{500} = \frac{3}{5}</math>. The largest possible two-day success ratio that Beta could achieve is <math> m/n, </math> where <mat
    9 KB (1,434 words) - 13:34, 29 December 2021
  • Dividing the two equations yields ...\theta</math>, so <math>\angle B'EA = \pi-2\theta</math>. Then <math>\tan(\pi-2\theta)=\frac{15}{8}</math>, or
    9 KB (1,501 words) - 05:34, 30 October 2023
  • ...to the smaller can be expressed in the form <math> \frac{a\pi+b\sqrt{c}}{d\pi-e\sqrt{f}}, </math> where <math> a, b, c, d, e, </math> and <math> f </math ...; the rest of the area of the circle is then equal to <math>\frac{2}{3}r^2\pi</math>.
    2 KB (329 words) - 23:20, 4 July 2013
  • ...to the smaller can be expressed in the form <math> \frac{a\pi+b\sqrt{c}}{d\pi-e\sqrt{f}}, </math> where <math> a, b, c, d, e, </math> and <math> f </math ...es and 10 blue candies. Terry picks two candies at random, then Mary picks two of the remaining candies at random. Given that the probability that they ge
    9 KB (1,410 words) - 05:05, 20 February 2019
  • ...llows the straight path that produces the earliest possible meeting of the two skaters, given their speeds. How many meters does Allie skate before meetin pair A=(0,0),B=(10,0),C=6*expi(pi/3);
    7 KB (1,045 words) - 20:47, 14 December 2023
  • ...th>x^{}_{}</math> satisfy the equation <math>\frac{1}{5}\log_2 x = \sin (5\pi x)</math>? Two three-letter strings, <math>aaa^{}_{}</math> and <math>bbb^{}_{}</math>, ar
    7 KB (1,106 words) - 22:05, 7 June 2021
  • ...is called ascending if, in its decimal representation, there are at least two digits and each digit is less than any digit to its right. How many ascendi In Pascal's Triangle, each entry is the sum of the two entries above it. The first few rows of the triangle are shown below.
    8 KB (1,117 words) - 05:32, 11 November 2023
  • ...the player's hand. The game ends if the player ever holds three tiles, no two of which match; otherwise the drawing continues until the bag is empty. Th ...\,</math> determine the number of times the light beam will bounce off the two line segments. Include the first reflection at <math>C\,</math> in your co
    7 KB (1,141 words) - 07:37, 7 September 2018
  • ...o sides of square <math>S_{i+1},</math> are the perpendicular bisectors of two adjacent sides of square <math>S_{i+2}.</math> The total area enclosed by ...<math>9</math> has a chord that is a common external tangent of the other two circles. Find the square of the length of this chord.
    6 KB (1,000 words) - 00:25, 27 March 2024
  • Given that <math>A_k = \frac {k(k - 1)}2\cos\frac {k(k - 1)\pi}2,</math> find <math>|A_{19} + A_{20} + \cdots + A_{98}|.</math> Except for the first two terms, each term of the sequence <math>1000, x, 1000 - x,\ldots</math> is o
    7 KB (1,084 words) - 02:01, 28 November 2023
  • ...ion bounded by consecutive circles is colored either red or green, with no two adjacent regions the same color. The ratio of the total area of the green r ...math> \mathcal{S}, </math> she writes on her list the greater of the set's two elements. Find the sum of the numbers on the list.
    6 KB (965 words) - 16:36, 8 September 2019
  • ...e1,2,3,\ldots,10\rbrace</math> Let <math>n</math> be the number of sets of two non-empty disjoint subsets of <math>\mathcal{S}</math>. (Disjoint sets are ...math> radians are <math>\frac{m\pi}{n-\pi}</math> and <math>\frac{p\pi}{q+\pi}</math>, where <math>m</math>, <math>n</math>, <math>p</math>, and <math>q<
    7 KB (1,177 words) - 15:42, 11 August 2023
  • ...gers is 6 times their sum, and one of the integers is the sum of the other two. Find the sum of all possible values of <math>N</math>. ...e log. The number of cubic inches in the wedge can be expressed as <math>n\pi</math>, where n is a positive integer. Find <math>n</math>.
    7 KB (1,127 words) - 09:02, 11 July 2023
  • The adjoining figure shows two intersecting chords in a circle, with <math>B</math> on minor arc <math>AD< ...ally, the circle <math>P</math> can intersect the chord <math>BC</math> at two points, one point, or they may not have a point of intersection. By the pro
    19 KB (3,221 words) - 01:05, 7 February 2023
  • <cmath>f(x) = \sin \frac{\pi x}{10}\sin \frac{\pi (x-4)}{10}</cmath> ...tice that it starts at x=0, then it goes to x=5, x=10, etc... each f() has two possible x values, but we are only counting the total number of x values so
    3 KB (588 words) - 14:37, 22 July 2020
  • Two solutions follow from here: ...}</math> and add them up before dividing by <math>3^7</math>. Here we have two ways to proceed:
    17 KB (2,837 words) - 13:34, 4 April 2024
  • ...ha + \beta</math> [[radian]]s, respectively, where <math>\alpha + \beta < \pi</math>. If <math>\cos \alpha</math>, which is a [[positive]] [[rational num pair O = (0,0), A = r*expi(pi/3);
    5 KB (763 words) - 16:20, 28 September 2019
  • Two solutions follow from here: ...eta</math> is the argument of <math>N</math> such that <math>0\leq\theta<2\pi.</math>
    7 KB (965 words) - 10:42, 12 April 2024
  • Solving these two equations, we find <math>x = \frac{-3x' + 4y'}{5}</math> and <math>y = \fra Now to find the equation of the hyperbola, we multiply the two expressions together to get one side of the equation: <math>(3x-4y)(4x+3y)=
    4 KB (700 words) - 17:21, 3 May 2021
  • ...s the [[straight]] path that produces the earliest possible meeting of the two skaters, given their speeds. How many meters does Allie skate before meetin pair A=(0,0),B=(10,0),C=6*expi(pi/3);
    5 KB (864 words) - 19:55, 2 July 2023
  • ...method involves drawing a triangle connecting the center of the 12-gon to two vertices of the 12-gon. Since the distance from the center to a vertex of To simplify the two nested radicals, add them, and call the sum <math>x</math>:
    6 KB (906 words) - 13:23, 5 September 2021
  • ...8</math> and <math>48</math> is <math>144</math>, so define <math>n = e^{2\pi i/144}</math>. We can write the numbers of set <math>A</math> as <math>\{n^ ...</math> are relatively prime, and by the [[Chicken McNugget Theorem]], for two relatively prime integers <math>a,b</math>, the largest number that cannot
    3 KB (564 words) - 04:47, 4 August 2023
  • .../math> for <math>1 \le i \le n</math> and <math>0 \le \theta_{i} < \frac {\pi}{2}</math>. We then have that ...e can see that it is basically calculating distance between the origin and two points. Therefore we can assume every <math>\sqrt{(2k-1)^2+a_k^2}</math> to
    4 KB (658 words) - 16:58, 10 November 2023
  • ...The sum of the areas of the twelve disks can be written in the from <math>\pi(a-b\sqrt{c})</math>, where <math>a,b,c^{}_{}</math> are positive integers a dot((cos(i*pi/6), sin(i*pi/6)));
    4 KB (740 words) - 19:33, 28 December 2022
  • ...^{}_{}</math> satisfy the [[equation]] <math>\frac{1}{5}\log_2 x = \sin (5\pi x)</math>? ...h>y = 0</math>, there is exactly <math>1</math> touching point between the two functions: <math>\left(\frac{1}{5},0\right)</math>. When <math>y < 0</math>
    2 KB (300 words) - 16:01, 26 November 2019
  • ...area of the two circles and then subtracting out their overlap. There are two methods of finding the area of overlap: ...of those three graphs is <math>40^2-(200\pi + 400) \Rightarrow 1200 - 200\pi \approx 571.68</math>
    2 KB (323 words) - 12:05, 16 July 2019
  • <math> \mathrm{(A) \ } \frac{\pi}{3}+1-\sqrt{3}\qquad \mathrm{(B) \ } \frac{\pi}{2}(2-\sqrt{3}) ...rm{(D) \ } \frac{\pi}{6}+\frac{\sqrt{3}+1}{2}\qquad \mathrm{(E) \ } \frac{\pi}{3}-1+\sqrt{3} </math>
    5 KB (873 words) - 15:39, 29 May 2023
  • ...points of <math>\triangle ABC\,</math> can be written in the form <math>q\pi-r\sqrt{s},\,</math> where <math>q, r,\,</math> and <math>s\,</math> are pos ...of the locus of <math>P</math> (shaded region below) is simply the sum of two [[segment]]s of the circles. If we construct the midpoints of <math>M_1, M_
    4 KB (717 words) - 22:20, 3 June 2021
  • ...\,</math> determine the number of times the light beam will bounce off the two line segments. Include the first reflection at <math>C\,</math> in your co ...a * pi/180),N); path r = C + .4 * expi(beta * pi/180) -- C - 2*expi(beta * pi/180);
    2 KB (303 words) - 00:03, 28 December 2017
  • Using DeMoivre, <math>13 - t = \text{cis}\left(\frac {(2k + 1)\pi}{10}\right)</math> where <math>k</math> is an integer between <math>0</math ...pi}{10}\right) \Rightarrow \bar{t} = 13 - \text{cis}\left(-\frac {(2k + 1)\pi}{10}\right)</math>.
    3 KB (375 words) - 23:46, 6 August 2021
  • ...d the area of either of them can be expressed uniquely in the form <math>m\pi-n\sqrt{d},</math> where <math>m, n,</math> and <math>d_{}</math> are positi Let the center of the circle be <math>O</math>, and the two chords be <math>\overline{AB}, \overline{CD}</math> and intersecting at <ma
    3 KB (484 words) - 13:11, 14 January 2023
  • pair B=(0,0), A=expi(pi/4), C=IP(A--A + 2*expi(17*pi/12), B--(3,0)), D=A+C, O=IP(A--C,B--D); Dividing the two equalities yields
    5 KB (710 words) - 21:04, 14 September 2020
  • ...are not common to both equations/sets, or else we are overcounting a root two times, rather than once. Try out some equation to see where this might appl Now, we see the <math>z^2+z-1</math> and it reminds us of the sum of two cubes. Cleverly factoring, we obtain that..
    6 KB (1,022 words) - 20:23, 17 April 2021
  • Given that <math>A_k = \frac {k(k - 1)}2\cos\frac {k(k - 1)\pi}2,</math> find <math>|A_{19} + A_{20} + \cdots + A_{98}|.</math> ...uates to an integer ([[triangular number]]), and the [[cosine]] of <math>n\pi</math> where <math>n \in \mathbb{Z}</math> is 1 if <math>n</math> is even a
    1 KB (225 words) - 02:20, 16 September 2017
  • Let <math>x=e^{\frac{i\pi}{36}}</math>. By Euler's Formula, <math>\sin{5k^\circ}=\frac{x^k-\frac{1}{x We factor the <math>\frac{1}{2i}</math> and split into two geometric series to get <math>\frac{1}{2i}\left(\frac{-\frac{1}{x^{35}}(x^{
    4 KB (614 words) - 04:38, 8 December 2023
  • ...re positive, <math>z</math> lies in the first quadrant and <math>\theta < \pi/2</math>; hence by right triangle trigonometry <math>\sin \theta = \frac{\s ...point of <math>OP</math> is <math>(0.5c, 0.5d)</math>. Since the slopes of two respectively nonvertical and nonhorizontal lines have a product of <math>-1
    6 KB (1,010 words) - 19:01, 24 May 2023
  • ...the volume of the liquid can be found by <math>\frac{\pi}{3}r^2h - \frac{\pi}{3}(r')^2h'</math>. ...rac{\pi}{3}\left(\frac{3}{4}r\right)^2 \left(\frac{3}{4}h\right) &= \frac{\pi}{3}\left(r^2h - \left(\frac{rh'}{h}\right)^2h'\right)\\
    4 KB (677 words) - 16:33, 30 December 2023
  • ...ion bounded by consecutive circles is colored either red or green, with no two adjacent regions the same color. The [[ratio]] of the total area of the gre ...\ldots - 1^{2} \pi</math>, while the total area is given by <math>100^{2} \pi</math>, so the ratio is
    4 KB (523 words) - 15:49, 8 March 2021
  • <math>f + 4 i = (b + 2 i)\left(e^{i(2 \pi / 3)}\right) = (b + 2 i)\left(-1/2 + \frac{\sqrt{3}}{2} i\right) = -\frac{b The area of the hexagon can then be found as the sum of the areas of two congruent triangles (<math>EFA</math> and <math>BCD</math>, with height <ma
    9 KB (1,461 words) - 15:09, 18 August 2023
  • ...e log. The number of cubic inches in the wedge can be expressed as <math>n\pi</math>, where n is a positive integer. Find <math>n</math>. ...king it to the existing one). Thus, <math>V=\dfrac{6^2\cdot 12\pi}{2}=216\pi</math>, so <math>n=\boxed{216}</math>.
    1 KB (204 words) - 17:41, 30 July 2022
  • ...math> radians are <math>\frac{m\pi}{n-\pi}</math> and <math>\frac{p\pi}{q+\pi}</math>, where <math>m</math>, <math>n</math>, <math>p</math>, and <math>q< ...{\alpha}</math> are <math>\theta = \pi-\alpha</math>, and <math>\theta = 2\pi + \alpha</math>.
    2 KB (336 words) - 19:30, 24 June 2020
  • ...tball game was played between two teams, the Cougars and the Panthers. The two teams scored a total of 34 points, and the Cougars won by a margin of 14 po ...at any point, and the sides of the square are parallel to the sides of the two given rectangles. What is the smallest possible area of the square?
    14 KB (2,059 words) - 01:17, 30 January 2024
  • ...f five for &#36;<math>2</math>. They sell all the candy bars at a price of two for &#36;<math>1</math>. What was the profit, in dollars? ...rac{3\pi}{16} \qquad \mathrm{(D)} \frac{\pi}{4} \qquad \mathrm{(E)} \frac{\pi}{2} </math>
    12 KB (1,874 words) - 21:20, 23 December 2020
  • ...athrm{(D) \ } \frac{\sqrt{3}}2 - \frac{\pi}6 \qquad \mathrm{(E) \ } \frac{\pi}6 </cmath> ...\mathrm{(C) \ }\pi/3 \qquad \mathrm{(D) \ }2\pi/3 \qquad \mathrm{(E) \ }3/\pi/4 </cmath>
    14 KB (2,102 words) - 22:03, 26 October 2018
  • .../math>, <math>CA=1</math>, and <math>AB=3</math>. If <math>\angle A=\frac{\pi}{n}</math> where <math>n</math> is an integer, find the remainder when <mat ...{2}+\frac{101}{100}</math> and <math>P_{2}: x=y^{2}+\frac{45}{4}</math> be two parabolas in the cartesian plane. Let <math>\mathcal{L}</math> be the commo
    8 KB (1,355 words) - 14:54, 21 August 2020
  • ...ath> on the arc that are the farthest away from each other. Since <math>34\pi</math> is <math>1/3</math> of the circumference of a circle with radius <ma
    1 KB (231 words) - 18:10, 10 July 2014
  • ...h that the median drawn to the hypotenuse is the [[geometric mean]] of the two legs of the triangle. ...the triangle must be <math> \frac{ \pi }{12} </math> and <math> \frac{ 5 \pi }{12}</math>, which are not difficult to construct. Q.E.D.
    6 KB (939 words) - 17:31, 15 July 2023
  • ...ath> and <math>b</math>. Prove that the set <math>A</math> has ''exactly'' two elements. ...ngle with the base <math>BC</math>. We know that <math>\angle ABD = \frac{\pi}{2}</math>. Let <math>M</math> be the midpoint of <math>BC</math>. The poin
    11 KB (1,779 words) - 14:57, 7 May 2012
  • ...math> intersects it in 2 points and the tangent line at <math>\left(\frac{\pi}2, 1\right)</math> intersects it in [[infinite]]ly many points (and is in f ...y. Any two disjoint circles have four tangents in common, two internal and two external.
    2 KB (332 words) - 21:54, 11 March 2024
  • ...of every right angle is 90 [[degree (geometry) | degrees]] or <math>\frac \pi 2</math> [[radian]]s. When drawing diagrams, we denote right angles with a
    673 bytes (91 words) - 23:59, 11 June 2022
  • The '''integral''' is one of the two base concepts of [[calculus]], along with the [[derivative]]. In introductory, high-school level texts, the integral is often presented in two parts, the '''indefinite integral''' and '''definite integral'''. Although
    5 KB (909 words) - 14:16, 31 May 2022
  • An '''angle''' is the [[union]] of two [[ray]]s with a common [[endpoint]]. The common endpoint of the rays is ca If two angles are [[congruent (geometry)|congruent]], they have the same angle mea
    3 KB (568 words) - 20:21, 21 June 2019
  • ...>2</math> greater than <math>p</math>. What is the arithmetic mean of the two primes in the smallest twin prime pair? ...ath>4</math> years older than my youngest grandparent. Each grandfather is two years older than his wife. If Bertha is younger than Dolores, what is the d
    33 KB (5,177 words) - 21:05, 4 February 2023
  • ...e of every straight angle is 180 [[degree (geometry) | degrees]] or <math>\pi</math> [[radian]]s.
    568 bytes (78 words) - 13:50, 12 June 2022
  • Three tiles are marked <math>X</math> and two other tiles are marked <math>O</math>. The five tiles are randomly arranged There are two values of <math>a</math> for which the equation <math> 4x^2 + ax + 8x + 9 =
    14 KB (2,026 words) - 11:45, 12 July 2021
  • The sum of the two 5-digit numbers <math>AMC10</math> and <math>AMC12</math> is <math>123422</ ...e numbers is <math>20</math>. The first is four times the sum of the other two. The second is seven times the third. What is the product of all three?
    13 KB (1,900 words) - 22:27, 6 January 2021
  • ...CE</math> is divided at <math>C</math> in the ratio <math>2:3</math>. The two semicircles, <math>ABC</math> and <math>CDE</math>, divide the circular reg ...{2}\pi\cdot3^2 - \frac{1}{2}\pi\cdot2^2 =\frac{\pi}{2}(5^2 + 3^2 - 2^2)=15\pi</math>
    2 KB (394 words) - 17:05, 20 October 2023
  • ...es of arcs, <math> \angle DTA = \angle DCO = \frac{\angle DCB}{2} = \frac{\pi}{2} - \frac{\angle DAB}{2} </math>. It follows that <math>AT = AD </math>. ...</math> such that <math>AT = AD </math>. Then <math>\angle DTA = \frac{ \pi - \angle DAB}{2} = \angle DCO</math>, so <math>DCOT </math> is a cyclic qua
    4 KB (684 words) - 07:28, 3 October 2021
  • ...hat is the probability that the number of heads obtained from flipping the two fair coins is the same? ...tomato, lettuce, pickles, cheese, and onions. A customer can choose one, two, or three meat patties, and any collection of condiments. How many differe
    15 KB (2,092 words) - 20:32, 15 April 2024
  • ...rotated about the other leg, the volume of the cone produced is <math>1920\pi \;\textrm{cm}^3</math>. What is the length (in cm) of the hypotenuse of the ...alpha + \beta </math> radians, respectively, where <math>\alpha + \beta < \pi</math>. If <math>\cos \alpha</math>, which is a positive rational number, i
    7 KB (1,071 words) - 19:24, 23 February 2024
  • ...rotated about the other leg, the volume of the cone produced is <math>1920\pi \;\textrm{ cm}^3</math>. What is the length (in cm) of the [[hypotenuse]] o ...}b</math>. Then <math>\frac{1}{3} \pi \left(\frac{12}{5}b\right)b^2 = 800\pi</math> so <math>b^3 = 1000</math> and <math>b = 10</math> so <math>a = 24</
    3 KB (463 words) - 15:10, 4 September 2020
  • draw(2*expi(2*pi/6)--2*expi(4*pi/6)); ...}{4}+\frac{1}{24}\pi\qquad \mathrm{(E) \ } \frac{\sqrt{3}}{4}+\frac{1}{12}\pi </math>
    3 KB (380 words) - 21:53, 19 March 2022
  • A '''sector''' of a [[circle]] is a region bounded by two [[radius|radii]] of the circle and an [[arc]]. If the [[central angle]] of the sector is <math>\pi</math> (or <math>180^{\circ}</math>), then the sector is a [[semicircle]].
    1 KB (178 words) - 21:12, 24 April 2008
  • ...annual [[math tournament]] hosted and run by the School of Mathematics and Pi Mu Epsilon of the Georgia Institute of Technology (better known as Georgia After lunch, there is a two-hour, six-question proof test. These papers are graded by School of Mathema
    3 KB (475 words) - 21:51, 31 December 2013
  • There are two things that Asymptote users commonly would like to do with their figures: m \(\int_0^{\pi}{\sin{x}}\,dx=2\)
    12 KB (1,931 words) - 13:53, 26 January 2020
  • ...'' is associated with a cartesian coordinate pair in Asymptote. There are two useful functions that allow one to use polar coordinates as well: ...joined smoothly in an curve that can be twisted. Keep in mind that if only two things are connected with <tt>..</tt>, the curve will be a straight line.
    7 KB (1,205 words) - 21:38, 26 March 2024
  • .../math>; thus the functions is undefined at <math>x=\frac{\pi}{6} + \frac{n\pi}{3}</math>. ...ymptote is at the quotient of the leading coefficient of <math>P(x)</math> over the leading coefficient of <math>Q(x)</math>.
    4 KB (664 words) - 11:44, 8 May 2020
  • ...r centers is <math>84</math>, then the area of the third circle is <math>k\pi</math> for some integer <math>k</math>. Determine <math>k</math>. ...lish if and only if between any two <math>O's</math> there appear at least two consonants. Let <math>N</math> denote the number of <math>10</math>-letter
    7 KB (1,135 words) - 23:53, 24 March 2019
  • ...r centers is <math>84</math>, then the area of the third circle is <math>k\pi</math> for some integer <math>k</math>. Determine <math>k</math>. ...h>, so <math>r=14</math>. Thus the area of the circle is <math>\boxed{196}\pi</math>.
    795 bytes (129 words) - 10:22, 4 April 2012
  • ...the circle. More generally, an arc is a portion of a smooth curve joining two points. ..., in particular, the [[circumference]] of a circle is given by <math>C = 2\pi</math>.
    1 KB (170 words) - 00:24, 16 December 2023
  • ...t starts. The distance <math>x</math> can be expressed in the form <math>a\pi+b\sqrt{c},</math> where <math>a,</math> <math>b,</math> and <math>c</math> If it weren’t for the small tube, the larger tube would travel <math>144\pi</math>. Consider the distance from which the larger tube first contacts the
    2 KB (407 words) - 16:31, 29 February 2020
  • ...ers can produce <math>300</math> widgets and <math>200</math> whoosits. In two hours, <math>60</math> workers can produce <math>240</math> widgets and <ma ...t starts. The distance <math>x</math> can be expressed in the form <math>a\pi+b\sqrt{c},</math> where <math>a,</math> <math>b,</math> and <math>c</math>
    9 KB (1,435 words) - 01:45, 6 December 2021
  • The larger of two consecutive odd integers is three times the smaller. What is their sum? <math>\mathrm{(A)}\ 8.64\qquad \mathrm{(B)}\ 12\qquad \mathrm{(C)}\ 5\pi\qquad \mathrm{(D)}\ 17.28\qquad \mathrm{(E)}\ 18</math>
    11 KB (1,750 words) - 13:35, 15 April 2022
  • ...i/2)--intersectionpoint(incircle(A,B,C),I--I+O-A)-abs(A-O)*expi(angle(A-O)-pi/2), q_a--q_a+O-A); ...i/2)--intersectionpoint(incircle(A,B,C),I--I+A-O)-abs(A-O)*expi(angle(A-O)-pi/2), O--A);
    7 KB (1,274 words) - 15:11, 31 August 2017
  • ...] from a line [[perpendicular]] to both planes. Thus, a '''cylinder''' has two circular flat surfaces--think of a can of soup. Typically, to solve for sur * Surface [[area]]: <math>2 \pi r^2 + 2 \pi r h</math>
    827 bytes (135 words) - 18:13, 27 July 2023
  • ...ent pulleys, that are connected with a strap. If the distances between any two pulley center points are <math>AB=3\,\mathrm{m}</math>, <math>AC=4\,\mathrm ...\qquad \mathrm{(B) \ } (12 + \pi)\,\mathrm{m}\qquad \mathrm{(C) \ } (12+ 4\pi)\,\mathrm{m}\qquad \mathrm{(D) \ } \left(12+\frac\pi5\right)\,\mathrm{m}\qq
    1 KB (175 words) - 22:21, 23 July 2020
  • ...l prove the theorem for a cyclic hexagon, using directed angles mod <math>\pi </math>. '''Lemma.''' Let <math>\omega_1, \omega_2 </math> be two circles which intersect at <math>M, N </math>, let <math>AB </math> be a ch
    4 KB (712 words) - 21:57, 25 April 2020
  • ...the [[equation]] <math>\sin{x}=\sin{(nx)}</math> on the interval <math>[0,\pi]</math>. What is <math>\sum_{n=2}^{2007} F(n)</math>? Notice that the solutions are basically reflections across <math>x = \frac{\pi}{2}</math>.
    4 KB (588 words) - 14:40, 23 August 2023
  • pair A=(0,0),B=(-3^.5,-3),C=(3^.5,-3),D=13*expi(-2*pi/3),E1=11*expi(-pi/3),F=E1+D; Because the ratio of the area of two similar figures is the square of the ratio of the corresponding sides, <mat
    6 KB (1,033 words) - 02:36, 19 March 2022
  • touches its two neighbors and the larger circle. What is the radius of the ...in the list <math>1, 2, 3, 4, 5, \dots, 10000</math> which contain exactly two consecutive <math>9</math>'s such as <math>993, 1992</math> and <math>9929<
    9 KB (1,449 words) - 20:49, 2 October 2020
  • ...>\angle BOA</math> is a right angle. <math>A</math> and <math>B</math> are two points on the circumference of circle of radius <math>OA</math>. Find the a .../3),6sin(4*pi/3))--(0,0))--(-3,-3)--intersectionpoint((8cos(5*pi/6),8sin(5*pi/6))--(0,0),(-3,3)--(-3,-3))--cycle,gray);
    11 KB (1,738 words) - 19:25, 10 March 2015
  • <math>p</math> is the product of two <math>4</math>-digit numbers formed by the digits <math>1,2,3,4,5,6,7,8</ma <math>a</math> and <math>b</math> are two numbers that have prime factors <math>3</math> and <math>5</math> only. <ma
    11 KB (1,713 words) - 22:47, 13 July 2023
  • ...e radian measure of the acute angle formed by the two lines? (Note: <math>\pi</math> radians is <math>180</math> degrees.) ...\pi}{6} \qquad \mathrm{(D) \ } \frac{\pi}{5} \qquad \mathrm{(E) \ } \frac{\pi}{4} </math>
    3 KB (476 words) - 03:50, 23 January 2023
  • ...t response, and 1.5 points for each problem left unanswered. After looking over the 25 problems, Sarah has decided to attempt the first 22 and leave the la ...lengths <math>AB = 5</math>, <math>BC = 6</math>, and <math>AC = 7</math>. Two bugs start simultaneously from <math>A</math> and crawl along the sides of
    12 KB (1,814 words) - 12:58, 19 February 2020
  • The area of <math>T</math> may be computed in two different ways: <cmath>V_1 = \frac{1}{3}\pi R^2 h</cmath>
    7 KB (1,214 words) - 18:49, 29 January 2018
  • ...t[n]{x}=\sqrt[n]{|x|}\left(\cos\frac{\theta+2\pi k}{n}+i\sin\frac{\theta+2\pi k}{n}\right)</math> , where <math>k=0,1,2,...,n-1</math> and <math>x\in\mat ...h>\sqrt[4]{16}=\sqrt[4]{16}\left(\cos\frac{0+2\pi\cdot0}{4}+i\sin\frac{0+2\pi\cdot0}{4}\right)\implies\boxed{2}</math>
    3 KB (532 words) - 16:52, 20 May 2020
  • The larger of two consecutive odd integers is three times the smaller. What is their sum? ...005, <math>78</math> in 2006, and is <math>85</math> in 2007. Between what two consecutive years was there the largest percentage increase?
    13 KB (2,058 words) - 17:54, 29 March 2024
  • ...oint, it moves one point, and if it is on an even-numbered point, it moves two points. If the bug begins on point 5, after 1995 jumps it will be on point ...n attendance. A league official who knows the actual numbers attending the two games notes that:
    17 KB (2,387 words) - 22:44, 26 May 2021
  • ...is drawn parallel to the base <math>AB</math>, dividing the triangle into two equal areas. ...mean (average) of a set of <math>50</math> numbers is <math>38</math>. If two numbers, namely, <math>45</math> and <math>55</math>, are discarded, the me
    22 KB (3,345 words) - 20:12, 15 February 2023
  • ...of the area of the circle, which is <math>\frac{1}{2} (\pi \cdot 4^2) = 8\pi \approx \boxed{\textbf{(E) }25}</math>. ...e area of the circle satisfying <math>f(x)+f(y) \le 0</math>, or <math>16 \pi</math>.
    2 KB (365 words) - 14:48, 7 March 2022
  • A circle passes through the three vertices of an isosceles triangle that has two sides of length <math>3</math> and a base of length <math>2</math>. What is ...\frac{81}{32}\pi \qquad\textbf{(D) } 3\pi \qquad\textbf{(E) } \frac{7}{2}\pi</math>
    5 KB (851 words) - 22:02, 26 July 2021
  • Let <math>\, k_1 < k_2 < k_3 < \cdots \,</math> be positive integers, no two consecutive, and let <math>\, s_m = k_1 + k_2 + \cdots + k_m \,</math> for ...f the three given colors (red, blue, yellow), under the constraint that no two adjacent sides may be the same color. By making a sequence of such modifica
    2 KB (391 words) - 07:58, 19 July 2016
  • Heather compares the price of a new computer at two different stores. Store <math>A</math> offers <math>15\%</math> off the sti ...}\ \left(2\sqrt{3}+3\right)^2\pi\\\mathrm{(E)}\ 9\left(\sqrt{3}+1\right)^2\pi</math>
    14 KB (2,138 words) - 15:08, 18 February 2023
  • ...tions, where 1 is positive and 1 is negative. The absolute values of these two solutions are also both less than 2. This proves the base case. ...l}{2^n+1}\cdot\pi</math>. As we can choose the range <math>0\leq\theta\leq\pi</math> to ensure no duplications, we get that, upon rearranging, <math>0\le
    3 KB (596 words) - 16:19, 28 July 2015
  • We can make two observations from this: ...<math>z_n = 2e^{i\pi/6}z_{n-1}</math>, so <math>z_{n-1} = \frac{1}{2}e^{-i\pi/6}z_n</math>. This is the reverse transformation. We have
    5 KB (745 words) - 10:58, 9 December 2022
  • ...diagonal are to be added. What will be the positive difference between the two diagonal sums? ...- 2ax + b = 0</math> has two real solutions. What is the average of these two solutions?
    12 KB (1,838 words) - 16:52, 7 October 2022
  • ...diagonal are to be added. What will be the positive difference between the two diagonal sums? ...is steps. The pedometer records up to <math>99999</math> steps, then flips over to <math>00000</math> on the next step. Pete plans to determine his mileage
    14 KB (2,199 words) - 13:43, 28 August 2020
  • ...ngle entry. Each entry in any row after the top row equals the sum of the two entries diagonally above it in the row immediately above it. How many entr ...\arctan\frac {1}{4} + \arctan\frac {1}{5} + \arctan\frac {1}{n} = \frac {\pi}{4}.</cmath>
    9 KB (1,536 words) - 00:46, 26 August 2023
  • ...Rudolph bikes, but Jennifer takes a five-minute break at the end of every two miles. Jennifer and Rudolph begin biking at the same time and arrive at the Let <math>a = \pi/2008</math>. Find the smallest positive integer <math>n</math> such that
    7 KB (1,167 words) - 21:33, 12 August 2020
  • ...\arctan\frac {1}{4} + \arctan\frac {1}{5} + \arctan\frac {1}{n} = \frac {\pi}{4}.</cmath> Applying this to the first two terms, we get <math>\arctan{\dfrac{1}{3}} + \arctan{\dfrac{1}{4}} = \arctan
    3 KB (490 words) - 22:36, 28 November 2023
  • ...whose angle at the longer base <math>\overline{AD}</math> is <math>\dfrac{\pi}{3}</math>. The [[diagonal]]s have length <math>10\sqrt {21}</math>, and po pair A=(0,0), D=(4,0), B= A+2 expi(1/3*pi), C= D+2expi(2/3*pi), E=(-4/3,0), F=(3,0);
    4 KB (629 words) - 22:38, 28 November 2023
  • pair A=(0,0), D=(2*r, 0), N=(r,0), E=N+r*expi(74*pi/180); pair A=(0,0), D=(2*r, 0), N=(r,0), E=N+r*expi(74*pi/180);
    8 KB (1,338 words) - 23:15, 28 November 2023
  • ...efine a ''move'' for the particle as a counterclockwise rotation of <math>\pi/4</math> radians about the origin followed by a translation of <math>10</ma ...he position of the particle after a move from <math>P</math>, then we have two equations for <math>x'</math> and <math>y'</math>:
    5 KB (725 words) - 22:37, 28 January 2024
  • Consider the two triangles <math>\triangle ABC</math> and <math>\triangle PQR</math> shown i pair P=(7,0), Q=(14,0), R=P+7expi(pi/3), M=(10,1.2);
    7 KB (1,221 words) - 18:57, 3 July 2013
  • ...be an acute-angled triangle, and let <math>P</math> and <math>Q</math> be two points on side <math>BC</math>. Construct point <math>C_1 </math> in such a ...th>Q' =Q</math>. All our angles will be directed, and measured mod <math>\pi</math>.
    6 KB (1,117 words) - 01:17, 11 October 2021
  • <center><!-- NOTE: two different versions --><asy> size(120); ...ath> and height <math>h</math> has [[volume]] <math>V = \frac{1}{3} \cdot \pi r^2 \cdot h</math>. This is a special case of the general formula for the
    7 KB (1,128 words) - 20:12, 27 September 2022
  • This diagonal [[partition]]s <math>\mathcal{P}</math> into two regions <math>\mathcal{Q},\, \mathcal{R}</math>, and is the side of an isos int n = 17; real r = 1; real rad = pi/2;
    8 KB (1,318 words) - 12:37, 20 April 2022
  • ...by the absolute value of the difference between the numbers written at the two neighboring vertices. Prove that Bert can make a sequence of moves, after w label("$c$",expi(pi/3),(0,0),red);
    5 KB (739 words) - 13:39, 4 July 2013
  • ...nstitute (which also had organized the meeting), had selected the problems over the previous several months. They were led by Arthur Jaffe, the first direc ...thoroughly examine the solution. This committee would consist of at least two world-renowned mathematicians and at least one member of the SAB.
    13 KB (1,969 words) - 17:57, 22 February 2024
  • ...{120}</math>. How many dollars did the family save by buying a family pass over buying single day passes for every member of the family? The difference between two prime numbers is <math>11</math>. Find their sum.
    71 KB (11,749 words) - 01:31, 2 November 2023
  • ...? Calculate an approximate answer by using <math>\pi=3.14</math> or <math>\pi=22/7</math>. ...</math>. Adding, we get <math>\frac{8}{3}\pi+\frac{\pi}{6}+\frac{\pi}{6}=3\pi</math>. Then approximate.
    854 bytes (145 words) - 23:28, 21 October 2022
  • ...t Closet, the price of a coat is discounted 20% from its &#36;90.00 price. Two clerks, Jack and Jill, calculate the bill independently. Jack brings up &#3 The Little Twelve Basketball League has two divisions, with six teams in each division. Each team plays each of the oth
    13 KB (1,821 words) - 22:18, 5 December 2023
  • ..._{AB}, P_{BC}, P_{CD}, P_{DA}</math>. Van Aubel's Theorem states that the two line segments connecting opposite centers are perpendicular and equal lengt ...by the complex number <math>x</math>. Note that <math>\angle PAB = \frac{\pi}{4}</math> and that <math>PA = \frac{\sqrt{2}}{2}AB</math>, and similarly f
    2 KB (410 words) - 14:01, 4 March 2023
  • ...\pi \qquad \text{(C)}\ 2\pi \qquad \text{(D)}\ 3\pi \qquad \text{(E)}\ 3.5\pi</math> Using the digits 1, 2, 3, 4, 5, 6, 7, and 9, form 4 two-digit prime numbers, using each digit only once. What is the sum of the 4 p
    11 KB (1,733 words) - 11:04, 12 October 2021
  • ...d \mathrm{(C) \ } 6\pi\qquad \mathrm{(D) \ } 9\pi\qquad \mathrm{(E) \ } 12\pi </math> ...ne}\qquad \mathrm{(C) \ } \text{two}\qquad \mathrm{(D) \ } \text{more than two, but finitely many}\qquad \mathrm{(E) \ } \text{infinitely many} </math>
    10 KB (1,540 words) - 22:53, 19 December 2023
  • ...ase that measures one yard on each side. He is tethered to a vertex with a two-yard rope. What is the area, in square yards, of the region outside of the ...i \qquad \text{(C)}\ 5\pi/2 \qquad \text{(D)}\ 8\pi/3 \qquad \text{(E)}\ 3\pi</math>
    1 KB (204 words) - 20:45, 28 May 2023
  • How many two-digit positive integers have at least one <math>7</math> as a digit? ...75+100\pi \qquad \mathrm{(D) \ } 100+100\pi \qquad \mathrm{(E) \ } 100+125\pi </math>
    13 KB (1,988 words) - 23:06, 7 March 2024
  • Two boundary points of a ball of radius 1 are joined by a curve contained in th A father, mother and son hold a family tournament, playing a two person board game with no ties. The tournament rules are:
    3 KB (427 words) - 18:55, 3 July 2013
  • ...ircle. Prove that the distance <math>d</math> between the centers of these two circles is ...cdot IL = PI\cdot QI</math> by Power of a Point. Therefore, <math>2r\rho = PI \cdot QI = (PO + OI)(QO - OI) = (r + d)(r - d)</math>, and we have <math>2r
    2 KB (308 words) - 06:29, 16 December 2023
  • ...ath>-axis and the entire shape is above the <math>x</math>-axis. There are two cases: the number of sides is even, and the number of sides is odd: ...h>y\textrm{-coordinate} = \sum_{k = 1}^{\frac{n}{2}}a_k \cdot \sin \frac{2\pi(k-1)}{n}.\textbf{ (1)}</cmath>
    4 KB (836 words) - 17:58, 7 December 2022
  • Prove that <math>\cos{\frac{\pi}{7}}-\cos{\frac{2\pi}{7}}+\cos{\frac{3\pi}{7}}=\frac{1}{2}</math>. ...tter. Exactly two of the contestants finished in the places predicted, and two disjoint pairs of students predicted to finish consecutively actually did s
    2 KB (342 words) - 21:16, 20 August 2020
  • The radius of a circle that has an area of <math>\frac{\pi}{\sqrt{2}}</math> is <math>r</math>. Find <math>r^{2}</math> A three digit natural number is <math>Alternative</math> if it has two even digits and one odd digit as its number digits. Find the number of alte
    11 KB (1,695 words) - 14:33, 7 March 2022
  • The large circle has diameter <math>\text{AC}</math>. The two small circles have their centers on <math>\text{AC}</math> and just touch a The small circle has radius <math>1</math>, thus its area is <math>\pi</math>.
    2 KB (277 words) - 21:32, 3 July 2013
  • ...in the picture, that is outside the smaller circle and inside each of the two larger circles? \mathrm{(A) \ } \frac{5}{3} \pi - 3\sqrt 2
    4 KB (703 words) - 22:37, 2 November 2022
  • An annulus is the region between two concentric circles. The concentric circles in the figure have radii <math>b ...hrm{(C) \ } \pi c^2 \qquad \mathrm{(D) \ } \pi d^2 \qquad \mathrm{(E) \ } \pi e^2 </math>
    2 KB (340 words) - 14:35, 23 April 2023
  • ...>(6m,0)</math>, and the line <math>y = mx</math> divides the triangle into two triangles of equal area. What is the sum of all possible values of <math>m< ...^3 + \cdots</math> and <math>a + ar_2 + ar_2^2 + ar_2^3 + \cdots</math> be two different infinite geometric series of positive numbers with the same first
    13 KB (2,105 words) - 13:13, 12 August 2020
  • \mathrm{(B)}\ \frac{\pi}{6} \mathrm{(C)}\ \frac{2}{\pi}
    14 KB (2,130 words) - 11:32, 7 November 2021
  • The first two terms of a sequence are <math>a_1 = 1</math> and <math>a_2 = \frac {1}{\sqr As the number of possible consecutive two terms is finite, we know that the sequence <math>b_n</math> is periodic. Wr
    7 KB (990 words) - 07:23, 24 October 2022
  • <center><cmath>p(2009 + 9002\pi i) = p(2009) = p(9002) = 0</cmath></center> From the three zeroes, we have <math>p(x) = (x - (2009 + 9002\pi i))(x - 2009)(x - 9002)</math>.
    2 KB (322 words) - 10:25, 29 July 2020
  • pair D=MP("D",(0,0)),C=MP("C",(12,0)),A=MP("A",C+14*expi(145*pi/180),N),B=MP("B",A+(9,0),N),E=IP(A--C,B--D);MP("9",(A+B)/2,N);MP("12",(C+D) ...\triangle AEB \sim \triangle EDC</math> (which should be trivial given the two equal triangles) we have that
    3 KB (543 words) - 21:09, 23 October 2023
  • ...Bethany did the same with a regular heptagon (7 sides). The areas of the two regions were <math>A</math> and <math>B</math>, respectively. Each polygon ...isosceles triangle formed by the center of the polygon <math>S</math> and two consecutive vertices <math>X</math> and <math>Y</math>. We are given that <
    4 KB (630 words) - 21:27, 30 December 2023
  • ...e area of the large circle is <math>L = \pi R^2 = \pi r^2 (1+\sqrt 2)^2 = \pi r^2 (3+2\sqrt 2)</math>. The area of four small circles is <math>S = 4\pi r^2</math>. Hence their ratio is:
    3 KB (474 words) - 12:50, 29 September 2023
  • ...h> as it rolls once around the circumference of circle <math>A</math>. The two circles have the same points of tangency at the beginning and end of circle ...cumference of circle <math>B</math> with radius <math>r</math> is <math>2r\pi</math>. Since circle <math>B</math> makes a complete revolution and ''ends
    2 KB (276 words) - 09:57, 8 June 2021
  • ...^\circ</math> sector of a circle of radius <math>10</math> by aligning the two straight sides? The length of the red line is <math>\dfrac{252}{360}\cdot 2\pi \cdot 10 = 14\pi</math>. This is the circumference of a circle with radius <math>7</math>.
    2 KB (279 words) - 00:32, 30 December 2023
  • A rectangular yard contains two flower beds in the shape of congruent isosceles right triangles. The remain Kiana has two older twin brothers. The product of their ages is <math>128</math>. What is
    13 KB (2,030 words) - 03:04, 5 September 2021
  • ...34 + \frac34i</math> into polar form as <math>\frac{3\sqrt{2}}{4}e^{\frac{\pi}{4}i}</math>. Restated using geometric probabilities, we are trying to find ...ath>2</math> by <math>2</math> square centered at the origin. Graphing our two equations gives us the four lines <cmath>x-y=\frac{4}{3},</cmath> <cmath>x-
    2 KB (422 words) - 13:25, 20 January 2020
  • For how many values of <math>x</math> in <math>[0,\pi]</math> is <math>\sin^{ - 1}(\sin 6x) = \cos^{ - 1}(\cos x)</math>? ...in^{-1}(x) \leq \pi/2</math> and <math>\forall x: 0\leq \cos^{-1}(x) \leq \pi</math>.
    7 KB (1,287 words) - 07:09, 22 December 2022
  • ...>O</math> be center of the circle and <math>P</math>,<math>Q</math> be the two points of tangent such that <math>P</math> is on <math>BI</math> and <math> Since the ratios between corresponding lengths of two similar diagrams are equal, we can let <math>AD = 144, CD = 420</math> and
    12 KB (1,970 words) - 22:53, 22 January 2024
  • ...the radius of the smaller circle is <math>x</math>, so it's area is <math>\pi x^2</math>. ...be <math>2x^2 \pi</math>. Putting one over the other and dividing, you get two as the answer: or <math>\boxed{(B)}</math>.
    1 KB (191 words) - 22:09, 14 January 2018
  • <cmath> \lvert f'(z_0) \rvert = \biggl\lvert \frac{1}{2\pi i} point <math>z \in \mathbb{C}</math>. Now for any two complex numbers <math>A</math>
    2 KB (412 words) - 20:30, 16 January 2024
  • ...e numbers lie in the interval between <math>\frac{5}{3}</math> and <math>2\pi?</math> Consider these two geoboard quadrilaterals. Which of the following statements is true?
    15 KB (2,165 words) - 03:32, 13 April 2024
  • ...ath> instead of <math>B</math>. <math>\angle AOC_1</math> = <math>\frac {\pi}{7}</math>. Using the [[Law of Cosines]], <math>\overline {AC_1}^2</math> = <math>8 - 8 \cos \frac {\pi}{7}</math>,
    8 KB (1,266 words) - 20:27, 10 December 2023
  • Determine all values <math>x</math> in the interval <math>0\leq x\leq 2\pi </math> which satisfy the inequality .../math> is equal to <math>k</math>. Compute the ratio of the volumes of the two solids obtained.
    3 KB (497 words) - 12:39, 29 January 2021
  • ...example, the fundamental group of a figure eight is the [[free group]] on two [[generator]]s, which is not abelian. However, the fundamental group of a c We say that two binary operations <math>\circ, \cdot</math> on a
    8 KB (1,518 words) - 20:11, 23 January 2017
  • ...ave to place the third point is a <math>60</math> degrees arc(if the first two are <math>60</math> degrees apart), with a <math>\frac{1}{6}</math> probabi ...ave to place the third point is a <math>120</math> degree arc(if the first two are the same point), with a <math>\frac{1}{3}</math> probability.
    4 KB (635 words) - 11:46, 1 September 2022
  • ...}\ \frac{0.4}{\pi} \qquad \textbf{(C)}\ 0.4 \qquad \textbf{(D)}\ \frac{4}{\pi} \qquad \textbf{(E)}\ 4</math> In a magical swamp there are two species of talking amphibians: toads, whose statements are always true, and
    12 KB (1,817 words) - 15:00, 12 August 2020
  • The image below shows the two curves for <math>k=4</math>. The blue curve is <math>x^2+y^2=k^2</math>, wh Clearly the only such integer is <math>k=1</math>, hence the two curves are only disjoint for <math>k=1</math> and <math>k=-1</math>.
    9 KB (1,622 words) - 20:53, 11 September 2023
  • ...math>\triangle ABC</math>, we get that <math>AC^2 = r^2+1^2-2r\cos{\frac{2\pi}{3}} = r^2+r+1</math>. Therefore, the area of <math>\triangle ACE</math> is Note: To verify that the quadratic <math>r^2-6r+1</math> has two positive roots, we can either solve for the roots directly or note that dis
    4 KB (690 words) - 10:13, 14 October 2022
  • ...\left(\sin(\pi x) \cdot \sin(2 \pi x) \cdot \sin (3 \pi x) \cdots \sin(8 \pi x)\right)</math>. The intersection of the domain of <math>f(x)</math> with ...align*}\sin(\pi x) \cdot \sin(2 \pi x) \cdot \sin (3 \pi x) \cdots \sin(8 \pi x) &= 0\\
    9 KB (1,434 words) - 17:54, 17 August 2022
  • The area of a circle whose circumference is <math>24\pi</math> is <math>k\pi</math>. What is the value of <math>k</math>? ...}\ \frac{0.4}{\pi} \qquad \textbf{(C)}\ 0.4 \qquad \textbf{(D)}\ \frac{4}{\pi} \qquad \textbf{(E)}\ 4</math>
    13 KB (1,902 words) - 11:20, 5 March 2023
  • Markala attended two meetings during her <math>9</math>-hour work day. The first meeting took <m \textbf{(A)}\ \dfrac{\pi}{3}-1
    12 KB (1,817 words) - 22:44, 22 December 2020
  • ...4)+shiftR--(n-1,-n+1)+r*expi(pi/4)+shiftR^^r*expi(5*pi/4)+shiftR--r*expi(5*pi/4)+(n-1,-n+1)+shiftR,linetype("4 4")); fill((n-1.5,-1.5) -- (n-1.5,-1.5)+r*expi(5.2*pi/6) -- (n-1.5,-1.5)+r*expi(3.3*pi/6) -- cycle); /* manual arrowhead? avoid resizing */
    55 KB (7,986 words) - 17:04, 20 December 2018
  • ...ll the positive divisors of <math>2010^2</math>. She then randomly selects two distinct divisors from this list. Let <math>p</math> be the probability tha Jackie and Phil have two fair coins and a third coin that comes up heads with probability <math>\fra
    8 KB (1,243 words) - 21:58, 10 August 2020
  • ...x = 15</math>, the [[volume]] of the resulting [[solid]] is <math>\frac {m\pi}{n\sqrt {p}}</math>, where <math>m</math>, <math>n</math>, and <math>p</mat ...about the line <math>y = x/3+5</math>, the resulting solid is the union of two right [[cone]]s that share the same base and axis.
    4 KB (636 words) - 16:46, 25 November 2023
  • ...ively. Line <math>\ell</math> divides region <math>\mathcal{R}</math> into two regions with areas in the ratio <math>1: 2</math>. Suppose that <math>AU = <math>Y = \frac {1}{3}\pi(3)^2 = 3\pi</math>
    10 KB (1,418 words) - 23:05, 20 October 2021
  • ...hooses one of the other four to shoot. The probability that there are some two people shooting each other can be expressed in the form <math>\frac{a}{b}</ ...r of arrangements, <math>4^5</math>. For any pair to shoot each other, the two shoot each other in exactly one way, while the other three people shoot any
    36 KB (6,214 words) - 20:22, 13 July 2023
  • LaTeX uses a special "math mode" to display mathematics. There are two types of this "math mode": Besides displaying in-line vs. displaying centered and on a new line, the two modes render differently in other ways. Note that <code>$\sum_{k=1}^n k^2$<
    8 KB (1,356 words) - 22:35, 26 June 2020
  • multiplying any two and dividing by the third, we get: &= \frac{\Gamma^4(\frac{1}{4})}{8\pi^2}
    11 KB (1,889 words) - 13:45, 4 July 2013
  • We know that angle <math>BIC = 135^{\circ}</math>, as the other two angles in triangle <math>BIC</math> add to <math>45^{\circ}</math>. Assume markangle(Label("$\scriptstyle{\frac{\pi}{4} - \frac{\theta}{2}}$"), radius=40, I, C, B);
    7 KB (1,230 words) - 19:47, 31 January 2024
  • ...rt will hit a given region is proportional to the area of the region. When two darts hit this board, the score is the sum of the point values of the regio ...> or <math>9\pi</math>. Note that the area of the whole circle is <math>36\pi</math>.
    3 KB (516 words) - 13:40, 23 January 2024
  • ...and either circle's center (symmetry, you chose!). The intersection of the two circles should form a geometrical lens shape. By sectors, <cmath>f(\theta)=\frac{\theta}{2\pi}\cdot2\pi=\theta</cmath>
    6 KB (1,105 words) - 13:39, 9 January 2024
  • Find the number of two-digit positive integers whose digits total <math>7</math>. ...of the five numbers in a list is <math>54</math>. The average of the first two
    13 KB (1,860 words) - 19:58, 8 May 2023
  • ...{4}</math> revolutions. In what direction does the spinner point after the two moves? The letter T is formed by placing two <math> 2 \times 4 </math> inch rectangles next to each other, as shown. Wha
    16 KB (2,215 words) - 19:18, 10 April 2024
  • Two circles lie outside regular hexagon <math>ABCDEF</math>. The first is tange ...area of the first circle is <math>(\frac{\sqrt{3}}{6})^2 \cdot \pi =\frac{\pi}{12}</math>.
    3 KB (455 words) - 19:28, 5 September 2022
  • ...have interior points in common. We paint in black the triangles that have two sides that are also sides of the polygon, in red if only one side of the tr Prove that there are two more black triangles that white ones.
    2 KB (451 words) - 01:03, 23 December 2018
  • A circle with center <math>O</math> has area <math>156\pi</math>. Triangle <math>ABC</math> is equilateral, <math>\overline{BC}</math The formula for the area of a circle is <math>\pi r^2</math> so the radius of this circle is <math>\sqrt{156}.</math>
    3 KB (439 words) - 13:21, 23 September 2023
  • ...c{\pi}{18} \qquad \textbf{(D)}\ \dfrac{1}{4} \qquad \textbf{(E)}\ \dfrac{2\pi}{9}</math> ...f by the two radii - area of the triangle with sides on the square and the two radii).
    3 KB (544 words) - 20:54, 24 March 2024
  • ...nts per text message sent, plus <math>10</math> cents for each minute used over <math>30</math> hours. In January Michelle sent <math>100</math> text messa ...ber of successful free throws was one more than their number of successful two-point shots. The team's total score was <math>61</math> points. How many fr
    13 KB (1,994 words) - 13:52, 3 July 2021
  • ...mes \frac{1}{6} \times \frac{1}{6} = \frac{2}{36}</math>. The sum of these two probabilities now gives the final answer: <math>\frac{1}{36} + \frac{2}{36}
    2 KB (248 words) - 14:51, 5 May 2021
  • Which of these four numbers <math> \sqrt{\pi^2},\,\sqrt[3]{.8},\,\sqrt[4]{.00016},\,\sqrt[3]{-1}\cdot \sqrt{(.09)^{-1}}< The sum of two numbers is <math> 10</math>; their product is <math> 20</math>. The sum of
    25 KB (3,872 words) - 14:21, 20 February 2020
  • ...math>¢ per text message sent, plus <math>10</math>¢ for each minute used over <math>30</math> hours. In January Juan sent <math>100</math> text messages ...ber of successful free throws was one more than their number of successful two-point shots. The team's total score was 61 points. How many free throws d
    13 KB (1,903 words) - 18:09, 19 April 2021
  • ...cmath>\tan (\theta) = \frac{3}{x}</cmath> YAY!!! We have two equations for two variables... that are relatively difficult to deal with. Well, we'll try to ...{6 - 3 \sqrt{3}} = 2 + \sqrt{3}</math>, <math>\theta = \frac{5 + 12n}{12} \pi</math>, where <math>n</math> is any integer. Converting to degrees, we have
    5 KB (782 words) - 14:29, 1 April 2024
  • ...went on a week-long trip together and agreed to share the costs equally. Over the week, each of them paid for various joint expenses such as gasoline and ...integers <math>a</math> and <math>b</math>, Ron reversed the digits of the two-digit number <math>a</math>. His erroneous product was 161. What is the c
    13 KB (1,978 words) - 16:28, 12 July 2020
  • ...d \mathrm{(C) \ } 6\pi\qquad \mathrm{(D) \ } 9\pi\qquad \mathrm{(E) \ } 12\pi </math> A line going through the centers of the two smaller circles also goes through the diameter. The length of this line wit
    2 KB (247 words) - 17:30, 5 January 2021
  • ...um of the numbers on every three consecutive vertices is a multiple of 3. Two acceptable arrangements are considered to be indistinguishable if one can b Suppose <math>x</math> is in the interval <math>[0,\pi/2]</math> and <math>\log_{24 \sin x} (24 \cos x) = \frac{3}{2}</math>. Fin
    10 KB (1,634 words) - 22:21, 28 December 2023
  • ...)}\ \pi \qquad \textbf{(D)}\ \frac{4\pi}{3} \qquad \textbf{(E)}\ \frac{5\pi}{3}</math> The curves of the track are semicircles, but since there are two of them, we can consider both of the at the same time by treating them as a
    2 KB (389 words) - 19:35, 7 August 2023
  • Suppose <math>x</math> is in the interval <math>[0, \pi/2]</math> and <math>\log_{24\sin x} (24\cos x)=\frac{3}{2}</math>. Find <ma There are now two ways to finish this problem.
    2 KB (330 words) - 20:47, 10 December 2023
  • ...7}}</math> are parallel, and its side length is the distance between these two lines. However, this is given to be the side length of the octagon, or <mat ...nt-and-a-line/ point to line distance formula], the distance between these two lines is <cmath>\frac{|c_{2}-c_{1}|}{\sqrt{a^{2}+b^{2}}}=\frac{2m\left(1+\s
    8 KB (1,344 words) - 18:39, 9 February 2023
  • ...the real parts of the blue dots is easily seen to be <math>8+16\cos\frac{\pi}{6}=8+8\sqrt{3}</math> and the negative of the sum of the imaginary parts o Note that <math>\sin(x) = \sin(x + \pi/2 - \pi/2) = \cos(x + \pi/2)</math>.
    5 KB (805 words) - 18:46, 27 January 2024
  • ...o went on a week-long trip together and agreed to share the costs equally. Over the week, each of them paid for various joint expenses such as gasoline and ...integers <math>a</math> and <math>b</math>, Ron reversed the digits of the two-digit number <math>a</math>. His erroneous product was <math>161</math>. Wh
    13 KB (2,090 words) - 18:05, 7 January 2021
  • ...l, and Al's pills cost a total of <math> \ </math><math>546</math> for the two weeks. How much does one green pill cost? ...math>. What is the distance between the <math>x</math>-intercepts of these two lines?
    15 KB (2,166 words) - 21:17, 16 February 2021
  • ...nd <math>1.5</math> points for each problem left unanswered. After looking over the <math>25</math> problems, Sarah has decided to attempt the first <math> Two points <math>B</math> and <math>C</math> are in a plane. Let <math>S</math>
    15 KB (2,297 words) - 12:57, 19 February 2020
  • ...(0,2).</math> What is the area of the intersection of the interiors of the two circles? ...\frac{\pi \sqrt{3}}{3} \qquad\textbf{(D) } 2(\pi -2) \qquad\textbf{(E) } \pi</math>
    898 bytes (142 words) - 20:42, 15 February 2024
  • The figure shown is the union of a circle and two semicircles of diameters <math>a</math> and <math>b</math>, all of whose ce The area of the whole circle is <math>A_{big} = \pi\cdot (a + b)^2</math>
    2 KB (307 words) - 18:58, 11 January 2014
  • ''Method 1:'' Dropping the altitude of our triangle splits it into two triangles. By HL congruence, these are congruent, so the "short side" is <m ...ac{ab\sin{C}}{2}</math>. Plugging in <math>a=b=s</math> and <math>C=\frac{\pi}{3}</math> (the angle at each vertex, in radians), we get the area to be <m
    1 KB (189 words) - 04:06, 18 June 2018
  • Solve for <math>x</math> for all answers in the domain <math>[0, 2\pi]</math>. We have a problem! The domain for values of <math>x</math> is <math>[0, 2\pi]</math>. However, no real value of <math>x</math> can become imaginary when
    8 KB (1,351 words) - 20:30, 10 July 2016
  • ...with <math>|z_0|=1.</math> What is the sum of all values <math>P(1)</math> over all the polynomials with these properties? ...these cases, <math>P(-1)=4-4+t-t+0=0</math>. The sum of <math>P(1)</math> over these cases is <math>\sum_{t=0}^{4} (4+4+t+t) = 40+20=60</math>.
    11 KB (1,979 words) - 17:25, 6 September 2021
  • The difference in the areas of two similar triangles is <math>18</math> square feet, and the ratio of the larg <math>\textbf{(A)}\ \frac{\pi m^2}{2}\qquad
    20 KB (3,108 words) - 14:14, 20 February 2020
  • The '''interior angle''' is the [[angle]] between two line segments, having two endpoints connected via a path, facing the path connecting them. ...erior angles of an <math>n</math> sided regular polygon,are <math>180(1-{2\over n})</math> degrees.
    673 bytes (105 words) - 22:28, 27 February 2020
  • ...qquad\textbf{(D) } \frac{\pi +\sqrt{3}}{2} \qquad\textbf{(E) } \frac{7}{6}\pi - \frac{\sqrt{3}}{2}</math> ...n into <math>\frac{5}{6}</math> of a circle with radius <math>1</math> and two equilateral triangles with side length <math>1</math>.
    4 KB (606 words) - 13:19, 9 July 2021
  • This looks a lot like the formula relating the slopes of two [[perpendicular]] [[Line|lines]], which is <math> m_1\times m_2=-1 </math>, ...> 2x+3x=5x </math>, so <math> 5x=\frac{\pi}{2} </math> and <math> x=\frac{\pi}{10} </math>, or <math> 18^\circ, \boxed{\text{A}} </math>.
    3 KB (493 words) - 18:16, 4 June 2021
  • ...mbered sectors. What is the probability that the sum of the numbers in the two sectors is prime? draw((0,0)--(cos(pi/6),sin(pi/6)));
    18 KB (2,551 words) - 18:46, 27 February 2024
  • ...\ 10+5\pi\qquad\text{(C)}\ 50\qquad\text{(D)}\ 50+5\pi\qquad\text{(E)}\ 25\pi </math> Draw two squares: one that has opposing corners at <math>A</math> and <math>B</math
    2 KB (383 words) - 16:58, 12 January 2024
  • ...angle. But <math>24</math> is too low, as it is less than the area of the two circles. Thus, the only reasonable answer is <math>\boxed{C}</math>, which
    2 KB (409 words) - 00:15, 5 July 2013
  • Two circles that share the same center have radii <math>10</math> meters and <m ...extbf{(C)}\ 10\pi+40\qquad\textbf{(D)}\ 20\pi+20\qquad \\ \textbf{(E)}\ 20\pi+40</math>
    2 KB (210 words) - 13:37, 19 October 2020
  • ...the center <math>O</math>. What is the ratio of the combined areas of the two semicircles to the area of circle <math>O</math>? ...ac{\sqrt 2}{4}\qquad\textbf{(B)}\ \frac{1}{2}\qquad\textbf{(C)}\ \frac{2}{\pi}\qquad\textbf{(D)}\ \frac{2}{3}\qquad\textbf{(E)}\ \frac{\sqrt 2}{2} </math
    2 KB (298 words) - 00:29, 18 December 2023
  • ...mbered sectors. What is the probability that the sum of the numbers in the two sectors is prime? draw((0,0)--(cos(pi/6),sin(pi/6)));
    1 KB (179 words) - 19:36, 15 April 2023
  • There are two ways to solve this problem. The first is more subtle, and the second is jus ...ngle POR,</math> and <math>b = \angle POQ.</math> Then <math>\angle ORP = \pi - x - a</math> and <math>\angle OQP = x - b.</math> Using the Law of Sines
    3 KB (568 words) - 12:24, 11 March 2018
  • ...-\pi\qquad\text{(C)}\ \frac{\pi} 2\qquad\text{(D)}\ \pi\qquad\text{(E)}\ 2\pi </math> Two opposite sides of a rectangle are each divided into <math>n</math> congruen
    15 KB (2,343 words) - 13:39, 19 February 2020
  • ...e, which is <math>9\pi</math>. The area of the shaded region is <math>36-9\pi</math> which less than <math>36-9(3)=9</math>, closest to <math>\boxed{\tex
    1 KB (191 words) - 00:10, 5 July 2013
  • If the digit <math>1</math> is placed after a two digit number whose tens' digit is <math>t</math>, and units' digit is <math Two boys <math>A</math> and <math>B</math> start at the same time to ride from
    22 KB (3,306 words) - 19:50, 3 May 2023
  • ...qquad \mathrm{(D) \ } \frac{5}{6}\pi+2 \qquad \mathrm{(E) \ }\frac{5}{3}\pi+2 </math> ...h> unit apart both horizontally and vertically. A rubber band is stretched over <math>4</math> pegs as shown in the figure, forming a quadrilateral. Its ar
    17 KB (2,488 words) - 03:26, 20 March 2024
  • ...qquad \mathrm{(D) \ } \frac{5}{6}\pi+2 \qquad \mathrm{(E) \ }\frac{5}{3}\pi+2 </math> .... Hence the total perimeter in cm is <math>\boxed{\text{(E)} \ \frac{5}{3}\pi+2}</math>.
    1 KB (181 words) - 18:35, 19 March 2024
  • ...ath>16</math> feet higher than the top of another tree. The heights of the two trees are in the ratio <math>3:4</math>. In feet, how tall is the taller tr ...\pi} \qquad\textbf{(C)}\ \pi \qquad\textbf{(D)}\ 2\pi \qquad\textbf{(E)}\ \pi^{2}</math>
    18 KB (2,768 words) - 21:05, 9 January 2024
  • ...s closest to the ratio of the circle's shaded area to the area between the two squares? ...area of the smaller square subtracted from the area of the circle: <math>\pi - 2.</math>
    3 KB (511 words) - 07:47, 22 May 2023
  • ...r with a tight rope. The rope is tight so that there is no extra rope left over when bound around the four circles. If the radius of a circle is <math>5</m Two triangles are combined together with overlap so that the Star of David is f
    15 KB (2,444 words) - 21:46, 1 January 2012
  • ...qquad \textbf{(D)}\ \dfrac{5}{2\pi}\text{ in} \qquad \textbf{(E)}\ \dfrac{\pi}{5}\text{ in}</math> ...(or else the ratio of the circumference to the diameter wouldn't be <math>\pi</math> anymore)
    2 KB (313 words) - 22:24, 24 December 2023
  • Two flag poles of height <math>h</math> and <math>k</math> are situated <math>2 ...l Angles. In effect, we can construct our desired point P by reflecting H' over IJ and finding the intersection of H'K and IJ. Therefore, there is only one
    1 KB (204 words) - 18:09, 19 September 2012
  • A square is cut into three rectangles along two lines parallel to a side, as shown. If the perimeter of each of the three r ...<math>100</math> does <math> x^{2}+x-n </math> factor into the product of two linear factors with integer coefficients?
    15 KB (2,247 words) - 13:44, 19 February 2020
  • A square with side length <math>8</math> is cut in half, creating two congruent rectangles. What are the dimensions of one of these rectangles? ...>4</math> times the reciprocal of the other number. What is the sum of the two numbers?
    13 KB (1,994 words) - 01:31, 22 December 2023
  • ...way. Arrange the five numbers in some order. Find the mean of the first two numbers, then find the mean of that with the third number, then the mean of ...as Chelsea. What is the probability that Alex wins three rounds, Mel wins two rounds, and Chelsea wins one round?
    14 KB (2,197 words) - 13:34, 12 August 2020
  • ...math> has its center <math>O</math> lying on circle <math>C_2</math>. The two circles meet at <math>X</math> and <math>Y</math>. Point <math>Z</math> in ...angle{OZY} \cong \angle{OZX}</math>, this is a system of two equations and two variables. Solving for <math>r</math> gives <math>r = \sqrt{30}</math>. <ma
    9 KB (1,496 words) - 02:40, 2 October 2022
  • ...\tan^2x-9\tan x+1=0</math> that are between <math>x=0</math> and <math>x=2\pi</math> radians. ...mathrm{(C) \frac{3\pi}{2} } \qquad \mathrm{(D) 3\pi } \qquad \mathrm{(E) 4\pi } </math>
    2 KB (282 words) - 21:14, 2 March 2019
  • ...uad \mathrm{(D) \frac{9}{\pi^2}(\sqrt{3}-1) } \qquad \mathrm{(E) \frac{9}{\pi^2}(\sqrt{3}+3) } </math> ...ac{6}{\pi}</math> for <math>r</math> and evaluating yields <math>\frac{9}{\pi^2}(\sqrt{3}+3)\implies{\boxed{E}}</math>.
    4 KB (649 words) - 10:04, 20 May 2021
  • ...y bags as possible, with 6 marbles per bag. How many marbles will be left over? ...e integers are added to the first two integers the sum is 41. Finally when two more integers are added to the sum of the previous four integers the sum is
    18 KB (2,350 words) - 18:48, 9 July 2023
  • ...along 2 radii to form 2 sectors, one with a central angle of 120. He makes two circular cones using each sector to form the lateral surface of each cone. ...h>, and the larger is <math>16\pi</math>. Those two arc lengths become the two circumferences of the new cones; so the radius of the smaller cone is <math
    2 KB (259 words) - 13:42, 2 September 2020
  • ...erline{AD}</math> has length <math>16</math>. What is the area between the two circles? ...quad\textbf{(C)}\ 64 \pi\qquad\textbf{(D)}\ 81 \pi\qquad\textbf{(E)}\ 100 \pi </math>
    2 KB (279 words) - 09:04, 10 March 2023
  • ...)}\ 2\qquad\textbf{(C)}\ \sqrt{5}\qquad\textbf{(D)}\ 3\qquad\textbf{(E)}\ \pi</math> ...gle inequality: <math>b<2a</math>. The only answer choice that holds these two inequalities is: <math>\sqrt{3}</math>.
    3 KB (432 words) - 20:18, 23 October 2021
  • ...the three sleepers were determined randomly, the probability that exactly two of the sleepers are from the same country is <math>\frac{m}{n}</math>, wher ...2</math> of the remaining <math>8</math> socks at random, and on Wednesday two of the remaining <math>6</math> socks at random. The probability that Wedn
    10 KB (1,615 words) - 21:48, 13 January 2024
  • ...= z,</math> and the imaginary part of <math>z</math> is <math>\sin{\frac{m\pi}{n}}</math>, for relatively prime positive integers <math>m</math> and <mat ...he plane divides the cube into two solids. The volume of the larger of the two solids can be written in the form <math>\tfrac{p}{q},</math> where <math>p<
    10 KB (1,617 words) - 14:49, 2 June 2023
  • ...ath> and <math>\sqrt{11}</math> from point <math>B</math>, so they are the two points where a circle centered at <math>A</math> with radius <math>\sqrt{11 There are two equilateral triangles with <math>\overline{AD_1}</math> as a side; let <mat
    13 KB (2,052 words) - 18:02, 5 February 2024
  • ...ves but doesn't know her house number. She tells him, "My house number has two digits, and exactly three of the following four statements about it are tru ...with congruent square tiles. If the total number of tiles that lie on the two diagonals is 37, how many tiles cover the floor?
    12 KB (1,771 words) - 21:13, 20 January 2024
  • ...math>, <math>\cdots</math>, <math>9\}</math> is called ''golden'' if every two consecutive numbers in the permutation are relatively prime. How many ''gol ...ing (possibly the same string) chosen at random. He continues on by tying two randomly selected untied ends together until there are no more untied ends,
    7 KB (1,309 words) - 11:13, 8 April 2012
  • if, for any two different points <math>A</math>, <math>B</math> in <math>\mathcal{S}</math> regular polygons <math>P_n</math> with <math>n</math> sides: given any two vertices <math>A</math> and
    4 KB (773 words) - 08:14, 19 July 2016
  • ...{a_n}</math> are all <math>n</math>th roots of unity. If <math>\omega=e^{2\pi i/n}</math>, then the sum of <math>32</math>nd powers of these roots will b ...mials of degree <math>n</math> over the field <math>\mathbb{F}_2</math> of two elements, and on the left, we have the factorisation of this generating fun
    8 KB (1,348 words) - 09:44, 25 June 2022
  • A geometric series is one where the ratio between each two consecutive terms is constant (ex. <math> 3, 6, 12, 24,\cdots </math>). The Given two <math> 2 </math>'s, "plus" can be changed to "times" without changing the r
    10 KB (1,477 words) - 16:02, 27 May 2012
  • ...e <math> M </math> minus the volume fo cone <math> N </math> is <math> 140\pi </math>, find the length of <math> \overline{BC} </math>. ...ath> S_n=\sum_{i=1}^{n-1}i\cdot S_{n-i} </math>. For example, if the first two elements are <math> 2 </math> and <math> 3 </math>, respectively, the third
    6 KB (910 words) - 17:32, 27 May 2012
  • ...s the square root of himself, or the square root of <math> i </math>. What two answers should he give? ...math> i=\cos\left(\frac{\pi}{2}\right)+i\sin\left(\frac{\pi}{2}\right)=e^{\pi i/2} </math>.
    1 KB (170 words) - 20:12, 27 May 2012
  • Two high school classes took the same test. One class of <math> 20 </math> stud Two equal circles in the same plane cannot have the following number of common
    23 KB (3,556 words) - 15:35, 30 December 2023
  • \textbf{(B)}\ \frac{440}{\pi} \qquad \textbf{(C)}\ \frac{880}{\pi} \qquad
    21 KB (3,123 words) - 14:24, 20 February 2020
  • The ratio of the areas of two concentric circles is <math>1: 3</math>. If the radius of the smaller is <m Two numbers whose sum is <math>6</math> and the absolute value of whose differe
    22 KB (3,509 words) - 21:29, 31 December 2023
  • ..., we can easily alternate them, but if <math>n</math> is odd, we must have two adjacent sides be the same length, and that length must be the larger side
    5 KB (871 words) - 18:59, 10 May 2023
  • ...xtbf{(C)}\ 12\pi+\sqrt{3}\qquad\textbf{(D)}\ 10\pi+9\qquad\textbf{(E)}\ 13\pi</math> ...<math>\dfrac{1}{2}</math>, we get <math>2\sqrt{3}</math>. Since there are two 30-60-90 triangles in the equilateral triangle, we multiply the area of the
    4 KB (675 words) - 03:58, 23 January 2023
  • Two distinct circles <math>K_1</math> and <math>K_2</math> are drawn in the pla ...lar to a line through a point on or off the line), find a construction for two points <math>C</math> and <math>D</math> on <math>K_1</math> such that <mat
    3 KB (500 words) - 19:00, 23 October 2019
  • ...(6) = 96\pi</math>. The volume of the wedge is half this which is <math>48\pi \approx \boxed{\textbf{(C)}\ 151}</math>.
    1 KB (198 words) - 20:54, 29 October 2023
  • In <math>2005</math> Tycoon Tammy invested <math>100</math> dollars for two years. During the the first year investment showed a <math>20\%</math> gain. Over the two-year period, what was the change
    14 KB (2,035 words) - 15:23, 26 January 2024
  • A rectangular photograph is placed in a frame that forms a border two inches wide on all sides of the photograph. The photograph measures 8 inche ...ass. Her goal is to achieve an average grade of 95 on the tests. Her first two test scores were 97 and 91. After seeing her score on the third test, she r
    13 KB (1,835 words) - 08:51, 8 March 2024
  • ...D)}\hspace{.05in}\frac{\pi-1}{\pi}\qquad\textbf{(E)}\hspace{.05in}\frac{3}\pi </math> ...two areas, we find the answer is <math> \boxed{\textbf{(A)}\ \frac{4-\pi}{\pi}} </math>.
    2 KB (310 words) - 16:18, 10 November 2023
  • ...rsecting sides, as shown. The circle's diameter is the segment between the two points of intersection. What is the area of the shaded region created by re ...uad \text{(C)}\ 28-4\pi \qquad \text{(D)}\ 28-2\pi \qquad \text{(E)}\ 32-2\pi</math>
    2 KB (346 words) - 21:43, 2 January 2023
  • ...m two sectors, the smaller having a central angle of 120 degrees. He makes two circular cones, using each sector to form the lateral surface of a cone. Wh ...s <math>\dfrac{1}{3} \times 4^2\pi \times 8\sqrt{2}=\dfrac{128\sqrt{2}}{3}\pi</math>.
    2 KB (364 words) - 00:51, 17 January 2021
  • <math>a_k = 2\sin ( \frac{k\pi}{12} )</math>. <math>2a_2 = 2 = a_6</math>. Thus any two segments with at least one them longer than <math>a_2</math> have a sum gre
    3 KB (395 words) - 15:54, 8 November 2022
  • ...on <math>20\%</math> of her three-point shots and <math>30\%</math> of her two-point shots. Shenille attempted <math>30</math> shots. How many points did ...operty that every term beginning with the third is the sum of the previous two. That is, <cmath> S_n = S_{n-2} + S_{n-1} \text{ for } n \ge 3. </cmath> S
    14 KB (2,204 words) - 20:25, 22 November 2020
  • ...One afternoon two of his brothers whose ages sum to 16 went to the movies, two brothers younger than 10 went to play baseball, and Joey and the 5-year-old ...on <math>20\%</math> of her three-point shots and <math>30\%</math> of her two-point shots. Shenille attempted <math>30</math> shots. How many points di
    12 KB (1,894 words) - 15:59, 3 January 2024
  • ...ath> P </math>, sweeping out a region whose area is <math> \frac{1}{c} (a \pi + b) </math>, where <math>a </math>, <math>b</math>, and <math> c </math> a ...h>. It must be that <math> \overline{AP} </math> divides the diagonal into two segments in the ratio <math>\sqrt{3}</math> to <math>1</math>. It is not di
    4 KB (603 words) - 16:51, 3 April 2020
  • ...e is the diameter, the opposite angle is a right angle. Also, because the two adjacent angles are one sixth of the circle apart, the angle opposite them ...ch arc is <math>\frac{\pi}{3}</math>, because the circumference is <math>2\pi</math>. Once we draw the triangle (as is explained in solution 1), we see t
    3 KB (417 words) - 20:21, 12 January 2024
  • ...into two squares and two nonsquare rectangles. The sum of the areas of the two squares is <math>\frac{9}{10}</math> of the area of square <math>ABCD.</mat ...uted in equal numbers to each student, there are exactly three blocks left over.
    9 KB (1,580 words) - 13:07, 24 February 2024
  • For <math>\pi \le \theta < 2\pi</math>, let ...>\sin \theta = -17/19, 1/3.</math> Since we're given <math>\pi\leq\theta<2\pi,</math> <math>\sin\theta</math> is nonpositive. We therefore use the negati
    10 KB (1,641 words) - 20:03, 3 January 2024
  • <math>|3-\pi|=</math> ...4 \qquad \textbf{(C) }3-\pi \qquad \textbf{(D) }3+\pi \qquad \textbf{(E) }\pi-3 </math>
    16 KB (2,451 words) - 04:27, 6 September 2021
  • ...here we require a clockwise rotation, so we multiply by <math>e^{-\frac{i\pi}{3}}</math> to obtain C. Upon averaging the coordinates of A, B, and C, we We construct point <math>C</math> by drawing two circles with radius <math>r = AB = \sqrt{13}</math>. One circle will be ce
    8 KB (1,268 words) - 14:10, 31 January 2024
  • A=expi(pi); B=expi(pi-a); C=expi(a); D=expi(0); E=expi(-a); F=expi(pi+a); ...342x^2-2420x=0</cmath>, it is easy to see that <math>x=-10, x=0</math> are two solutions for the equation, so we can factorize that into <cmath>x(x+10)(x^
    8 KB (1,357 words) - 09:23, 11 March 2024
  • ...de by attaching two equilateral triangles to the regular pentagon ABCDE in two of the five positions shown. How many non-congruent figures can be construc real r = 5/dir(54).x, h = 5 tan(54*pi/180);
    1 KB (237 words) - 23:06, 3 February 2020
  • Let <math>f(x)=(x^2+3x+2)^{\cos(\pi x)}</math>. Find the sum of all positive integers <math>n</math> for which First, let's split it into two cases to get rid of the absolute value sign
    3 KB (436 words) - 02:10, 12 December 2023
  • ...ted to put <math>z</math> in any other location.) Then there are precisely two possible distinct locations for <math>w</math>; one is obtained by going 12 Let the two possible locations for <math>w</math> be <math>W_1</math> and <math>W_2</ma
    6 KB (1,045 words) - 13:08, 21 January 2024
  • We see that both the radii are the two shorter sides of the triangle, making this a isosceles 45-45-90 triangle. We also see that the height that we drew is half the hypotenuse(note the two smaller 45-45-90 isosceles triangles).
    23 KB (3,182 words) - 12:30, 5 April 2014
  • <cmath>P = AI \cap \Omega \implies PB = PC = PI \implies</cmath> It is known that the composition of two axial symmetries with non-parallel axes is a rotation centered at
    15 KB (2,549 words) - 08:36, 2 September 2023
  • Kelvin the frog creates an infinite sequence of rational numbers. He chooses two starting terms <math>a_1=\tfrac{n_1}{d_1}</math> and <math>a_2=\tfrac{n_2}{ Kelvin the frog wants to find all positive integers <math>n</math> not over <math>9000</math> such that the order of <math>n \pmod{1000}</math> is <mat
    10 KB (1,710 words) - 23:23, 10 January 2020
  • ...thmetic progression. The inner rectangle is one foot wide, and each of the two shaded regions is <math>1</math> foot wide on all four sides. What is the l ...lobby to get some popcorn. When she returned, she found that Bea had moved two seats to the right, Ceci had moved one seat to the left, and Dee and Edie h
    14 KB (2,104 words) - 22:26, 16 September 2022
  • Let <math>A_0B_0C_0</math> and <math>A_1B_1C_1</math> be any two acute-angled triangles. Consider all triangles <math>ABC</math> that are si ...nt <math>P</math> inside <math>A_0B_0C_0</math> s.t. <math>\angle X_0PY_0=\pi-\angle X_1Z_1Y_1</math>, where <math>X,Y,Z</math> are a permutation of <mat
    2 KB (368 words) - 13:15, 29 January 2021
  • ...2,\ldots, w_m</math>. we claim that for some choice of <math>0\leq\theta<2\pi</math>, <math>v_i+e^{i\theta}w_j</math> will do the job(a suitable rotation ...y set and adding vectors to it, one by one. We just need to make sure that two thing are respected:
    4 KB (749 words) - 14:09, 29 January 2021
  • ...is the <math>3</math>-dimensional version of this), and then complete the two regular tetrahedra <math>A_1A_2A_3P_1,A_1A_2A_3P_2</math> having <math>A_1A ..._1,P_2</math>, these will be continuous functions of the angle that <math>\pi</math> makes with <math>\pi_i</math>, and for one of the points <math>P_1,P
    7 KB (1,370 words) - 15:42, 29 January 2021
  • <math>\textbf{(A)}\ \pi + 2 \qquad \textbf{(B)}\ \frac{2 \pi + 1}{2} \qquad
    17 KB (2,459 words) - 22:40, 10 April 2023
  • The number in each box below is the product of the numbers in the two boxes that touch it in the row above. For example, <math>30 = 6\times5</mat A fair coin is tossed 3 times. What is the probability of at least two consecutive heads?
    15 KB (2,162 words) - 20:05, 8 May 2023
  • A large rectangle is partitioned into four rectangles by two segments parallel to its sides. The areas of three of the resulting rectang each term is the sum of the two terms to its left. Find <math>a</math>.
    14 KB (2,124 words) - 13:39, 19 February 2020
  • ...D</math>. If <math>EB</math> has length one and <math>EC</math> has length two, then the area of the square is ...the smaller and the difference between the numbers is 8, the the larger of two numbers is
    17 KB (2,633 words) - 15:44, 16 September 2023
  • ...increase in circumference of a circle resulting from an increase in <math>\pi</math> units in the diameter. Then <math>P</math> equals: ...pi\quad\text{(C) } \frac{\pi^2}{2}\quad\text{(D) } \pi^2\quad\text{(E) } 2\pi</math>
    16 KB (2,571 words) - 14:13, 20 February 2020
  • ...morning for <math>\textdollar 2.50</math> each. In the afternoon she sells two thirds of what she has left, and because they are not fresh, she charges on The two legs of a right triangle, which are altitudes, have lengths <math>2\sqrt3</
    15 KB (2,190 words) - 15:21, 22 December 2020
  • ...qquad\textbf{(C)}\ 4\pi-4 \qquad\textbf{(D)}\ 2\pi+4 \qquad\textbf{(E)}\ 4\pi-2 </math> ...length <math>\sqrt{2}</math>. We deduce that the two triangles formed by two radii of circle <math>O</math> and the segment of the rectangle are 45-45-9
    2 KB (287 words) - 03:07, 28 May 2021
  • ...r{10,000}</math> the difference between a discount of <math>40</math>% and two successive discounts of <math>36</math>% and <math>4</math>%, Each of two angles of a triangle is <math>60^{\circ}</math> and the included side is <m
    21 KB (3,242 words) - 21:27, 30 December 2020
  • Two circles intersect at points <math>A</math> and <math>B</math>. The minor a <cmath>\dfrac{\pi x^2}{\pi y^2} = \dfrac{x^2}{y^2} </cmath>
    7 KB (1,191 words) - 23:37, 23 June 2022
  • ...consists of triangles whose sides have integer lengths less than 5, and no two elements of <math>S</math> are congruent or similar. What is the largest nu ...itive solutions of <cmath>2\cos(2x) \left(\cos(2x) - \cos\left( \frac{2014\pi^2}{x} \right)\right) = \cos(4x) - 1</cmath>
    13 KB (2,066 words) - 14:08, 1 November 2022
  • (one value of <math>a</math> which produces this maximum is <math>a=\frac{\pi}{4}</math>) ...^2+y^2</math>. Adding and subtracting the first two equations, dividing by two, and setting <math>\mu = 2\lambda</math> for convenience yields:
    4 KB (699 words) - 01:53, 30 April 2022
  • ...ee factors, the probability that a randomly selected man has exactly these two risk factors (but not the third) is 0.14. The probability that a randomly s ...es of length <math>a</math> can be pressed toward each other keeping those two sides parallel so the rectangle becomes a convex hexagon as shown. When the
    8 KB (1,410 words) - 00:04, 29 December 2021
  • Let <math>F</math> be the new tangency point of the two disks. The smaller disk rolled along minor arc <math>\overarc{AF}</math> on ...the smaller disk, so <math>6\alpha = 2\pi</math>, or <math>\alpha = \frac{\pi}{3}</math>.
    5 KB (854 words) - 20:02, 4 September 2021
  • ...ations of the even number 126, the largest possible difference between the two primes is Two congruent 30-60-90 are placed so that they overlap partly and their hypoten
    18 KB (2,788 words) - 13:55, 20 February 2020
  • If <math>3(4x+5\pi)=P</math> then <math>6(8x+10\pi)=</math> A square floor is tiled with congruent square tiles. The tiles on the two diagonals of the floor are black. The rest of the tiles are white. If there
    16 KB (2,548 words) - 13:40, 19 February 2020
  • Two circles are externally tangent. Lines <math>\overline{PAB}</math> and <math <math>\text{(A) } 1.44\pi\quad
    2 KB (295 words) - 19:09, 11 October 2016
  • This equation is <math>r^6e^{6\theta i}=2^6e^{(\pi\pm 2k\pi) i}</math>. Solving in the usual way, <math>r=2</math> and <math>\theta\in\ Thus there are only two solutions with positive real part, and they are conjugates, so their produc
    798 bytes (133 words) - 13:17, 4 February 2016
  • draw((0,1)--(cos(pi/14),-sin(pi/14))--(-cos(pi/14),-sin(pi/14))--cycle,dot); draw((-cos(pi/14),-sin(pi/14))--(0,-1/cos(3pi/7))--(cos(pi/14),-sin(pi/14)),dot);
    1 KB (221 words) - 14:09, 2 April 2023
  • draw((0,1)--(cos(pi/14),-sin(pi/14))--(-cos(pi/14),-sin(pi/14))--cycle,dot); draw((-cos(pi/14),-sin(pi/14))--(0,-1/cos(3pi/7))--(cos(pi/14),-sin(pi/14)),dot);
    14 KB (2,099 words) - 01:15, 10 September 2021
  • The area of the ring between two concentric circles is <math>12\tfrac{1}{2}\pi</math> square inches. The length of a chord of the larger circle tangent to <cmath>\pi a^2 - \pi b^2 = \frac{25 \pi}{2}</cmath>
    1 KB (187 words) - 03:29, 7 June 2018
  • The area of the ring between two concentric circles is <math>12\tfrac{1}{2}\pi</math> square inches. The length of a chord of the larger circle tangent to The arithmetic mean (ordinary average) of the fifty-two successive positive integers beginning at 2 is:
    16 KB (2,662 words) - 14:12, 20 February 2020
  • Two equal parallel chords are drawn <math>8</math> inches apart in a circle of <math>\textbf{(A)}\ 21\frac{1}{3}\pi-32\sqrt{3}\qquad
    2 KB (333 words) - 20:23, 15 April 2020
  • <math>\textbf{(A) }\frac{4}{\pi}\qquad \textbf{(B) }\frac{\pi}{\sqrt{2}}\qquad
    15 KB (2,366 words) - 07:52, 26 December 2023
  • ...eal numbers have the property that each number is the product of the other two? <math>\textbf{(A)}\ 36\pi \qquad
    16 KB (2,291 words) - 13:45, 19 February 2020
  • Using a table of a certain height, two identical blocks of wood are placed as shown in Figure 1. Length <math>r</ (A pair of socks is two socks of the same color. No sock may be counted in more than one pair.)
    17 KB (2,512 words) - 18:30, 12 October 2023
  • Consider the two functions <math>f(x) = x^2+2bx+1</math> and <math>g(x) = 2a(x+b)</math>, wh \textbf{(B)} \ \pi \qquad
    15 KB (2,309 words) - 23:43, 2 December 2021
  • If a number eight times as large as <math>x</math> is increased by two, then one fourth of the result equals <math>\textbf{(A)} \ \pi \qquad
    17 KB (2,500 words) - 19:05, 11 September 2023
  • <math>\textbf{(A) }\frac{\pi}{4}\qquad \textbf{(B) }\frac{3\pi}{4}\qquad
    17 KB (2,664 words) - 01:34, 19 March 2022
  • <math>\textbf{(A) }\frac{1}{\pi^2}\qquad \textbf{(B) }\frac{1}{\pi}\qquad
    15 KB (2,432 words) - 01:06, 22 February 2024
  • \textbf{(C) }\text{two}\qquad\\ \textbf{(D) }\text{a finite number greater than two}\qquad
    17 KB (2,835 words) - 14:36, 8 September 2021
  • A contractor estimated that one of his two bricklayers would take <math>9</math> hours to build a certain wall and the There are two positive numbers that may be inserted between <math>3</math> and <math>9</m
    17 KB (2,732 words) - 13:54, 20 February 2020
  • \textbf{(C) }\ 1: \pi \qquad \textbf{(D) }\ 3: \pi \qquad
    19 KB (2,873 words) - 18:57, 16 August 2023
  • The area of a circle inscribed in an equilateral triangle is <math>48\pi</math>. The perimeter of this triangle is: In the binary system, which has base two, the first five positive integers are <math>1,\,10,\,11,\,100,\,101</math>.
    26 KB (3,950 words) - 21:09, 31 August 2020
  • In the diagram, the two circles are tangent to the two parallel lines. The small circle is <math>9\pi</math>. Find the area of the shaded <math>\textit{lune}</math>, the region
    11 KB (1,648 words) - 09:55, 20 December 2021
  • In the diagram, the two circles are tangent to the two parallel lines. The (8)(6) - 2\left(\dfrac12 \cdot 3^2\pi\right) &= 48 - 2\left(\dfrac12 \cdot 9\pi\right) \\
    1 KB (162 words) - 14:13, 24 February 2022
  • region that lies between the two circles and the line. ...ac{1}{3} \cdot \pi - \frac{1}{6} \cdot 9\pi = \boxed {4\sqrt{3} - \frac{11\pi}{6}}</math>
    2 KB (336 words) - 12:57, 2 January 2020
  • ...hat every odd number <math>2n+1</math> can be expressed as a difference of two squares. (b) Demonstrate which even numbers can be expressed as a difference of two squares.
    6 KB (890 words) - 22:14, 7 November 2014
  • your answer as an exact multiple of <math>\pi</math> (and not as a ...triangle and let <math>A_1B_1</math> be the internal common tangent of the two largest circles, with the points <math>A_1</math> and <math>B_1</math> layi
    3 KB (412 words) - 18:49, 29 January 2018
  • pair A=(0,-1), E=(0,1), C=(0,0), D=dir(10), F=dir(190), B=(-1/sin(10*pi/180))*dir(10); ...for the equality of the two shaded areas, given <math>0 < \theta < \frac{\pi}{2}</math>, is
    2 KB (301 words) - 18:50, 1 April 2018
  • The sum of two prime numbers is 85. What is the product of these two prime numbers? ...ike from his house to Jill's house, which is located three blocks east and two blocks north of Jack's house. After biking each block, Jack can continue ei
    13 KB (1,957 words) - 12:08, 13 January 2024
  • ...}\frac{\pi}{5}\qquad\textbf{(D) }\frac{2\pi}{5}\qquad\textbf{(E) }\frac{2\pi}{3} </math> ...h> and time = distance/rate) to arrive at <math>\boxed{\textbf{(B) }\frac{\pi}{10}}</math> hours.
    2 KB (395 words) - 12:16, 17 January 2024
  • The sum of two positive numbers is <math> 5 </math> times their difference. What is the ra Two years ago Pete was three times as old as his cousin Claire. 2 years before
    12 KB (1,897 words) - 22:45, 18 March 2024
  • Two of the three sides of a triangle are 20 and 15. Which of the following numb The sum of two positive numbers is 5 times their difference. What is the ratio of the larg
    13 KB (2,117 words) - 12:33, 24 August 2023
  • ...e radius of the first. What is the relationship between the heights of the two cylinders? ...uting the first equation into the second and dividing both sides by <math>\pi</math>, we get <cmath>r_1^2h_1=\frac{121r_1^2}{100}h_2\implies h_1=\frac{12
    2 KB (258 words) - 23:04, 26 June 2023
  • ...tween the points is at least <math>\dfrac{1}{2}</math> is <math>\dfrac{a-b\pi}{c}</math>, where <math>a</math>, <math>b</math>, and <math>c</math> are po ...0.25 - x^2} dx = 4\left( \frac{1}{4} - \frac{\pi}{16} \right) = 1 - \frac{\pi}{4}.</cmath>
    8 KB (1,406 words) - 19:14, 9 October 2023
  • Points <math>( \sqrt{\pi} , a)</math> and <math>( \sqrt{\pi} , b)</math> are distinct points on the graph of <math>y^2 + x^4 = 2x^2 y + ...(C)} \ 2 \qquad\textbf{(D)} \ \sqrt{1+\pi} \qquad\textbf{(E)} \ 1 + \sqrt{\pi} </math>
    2 KB (418 words) - 15:25, 1 August 2022
  • ...t 6:00. It would already have completed three 180 degree turns. Therefore, two 180 degree turns would be completed at 4:00. ...s <math>20\pi</math>, so that is <math>20\pi</math> traveled on a <math>60\pi</math> arrow path. This is a ratio of 1/3, so the angle it carves is 120 de
    9 KB (1,380 words) - 09:00, 1 December 2023
  • <cmath>(\text{cos}(a\pi)+i\text{sin}(b\pi))^4</cmath> Let <math>\cos(a\pi) = x</math> and <math>\sin(b\pi) = y</math>. Consider the binomial expansion of the expression:
    5 KB (793 words) - 20:32, 26 May 2022
  • ...nce between the points is at least <math>\frac12</math> is <math>\frac{a-b\pi}{c}</math>, where <math>a,b,</math> and <math>c</math> are positive integer ...t{0.25 - x^2} dx = 4\left(\frac{1}{4} - \frac{\pi}{16}\right) = 1 - \frac{\pi}{4}.</cmath>
    12 KB (1,981 words) - 18:33, 3 September 2023
  • Isaac has written down one integer two times and another integer three times. The sum of the five numbers is <math ...week but never on two consecutive days. On Monday she plays basketball and two days later golf. She swims and plays tennis, but she never plays tennis the
    14 KB (2,247 words) - 11:38, 26 March 2024
  • Isaac has written down one integer two times and another integer three times. The sum of the five numbers is 100, ...c{\sqrt{2}}{2} \qquad\textbf{(D)}\; 4(\pi-\sqrt{3}) \qquad\textbf{(E)}\; 2\pi-\dfrac{\sqrt{3}}{2}</math>
    13 KB (2,064 words) - 13:39, 1 October 2022
  • ...c{\sqrt{2}}{2} \qquad\textbf{(D)}\; 4(\pi-\sqrt{3}) \qquad\textbf{(E)}\; 2\pi-\dfrac{\sqrt{3}}{2}</math> ...ction), so the answer is <math>4\pi-4\sqrt{3} = \boxed{\textbf{(D)}\; 4(\pi-\sqrt{3})}</math>.
    1 KB (197 words) - 15:02, 4 March 2015
  • ...ce on each half. The area of one of these unpainted faces is <math>a\cdot\pi + b\sqrt{c}</math>, where <math>a</math>, <math>b</math>, and <math>c</math ...\ + 12 \pi</math>. Thus, the area of section <math>ABCD</math> is <math>20\pi + 30\sqrt{3}</math>, and so our answer is <math>20+30+3=\boxed{053}</math>.
    9 KB (1,407 words) - 19:37, 17 February 2024
  • The first of these two options is shown in the image to the right. It is primarily useful for quic ...hat approximates how well a post correlates to the search query. The other two options, "Newest first" and "Oldest first", are self-explanatory.
    21 KB (3,334 words) - 23:53, 23 March 2022
  • 2) <math>3\pi</math> meets a blue triangle along the edge where the two faces meet. Consider the two triangles
    2 KB (310 words) - 00:19, 6 November 2015
  • ...at <math>(2,2)</math>. What is the area of the triangle enclosed by these two lines and the line <math>x+y=10</math>? ...s in a plane, there are exactly <math>N</math> distinct points that lie on two or more of the lines. What is the sum of all possible values of <math>N</ma
    14 KB (2,180 words) - 22:25, 25 April 2024
  • Isaac added two three-digit positive integers. All six digits in these numbers are differen ...the complement of the other. What is the sum of the degree measures of the two angles?
    12 KB (1,868 words) - 17:50, 30 December 2023
  • The harmonic mean of two numbers can be calculated as twice their product divided by their sum. The ...the complement of the other. What is the sum of the degree measures of the two angles?
    14 KB (2,037 words) - 19:09, 29 July 2023
  • ...te dreadful. EDIT: He now eats gnats again. He thinks that they taste like pi(e). EDIT: He ate 3,141,592,653,589,793,238,462,643,383 so far. 4. Gmaas wants every fact to have a pi reference. Gmaas created pi. EDIT: He made it 3.1415926 times.
    69 KB (11,805 words) - 20:49, 18 December 2019
  • ...face the New York Mets in the 2006 World Series. Assuming the two teams are evenly matched (each has a <math>.5</math> probability o \text{(N) }\pi\qquad
    31 KB (4,811 words) - 00:02, 4 November 2023
  • ...2} - \gamma.</math> Similarly, one can show that <math>\angle OAC = \frac{\pi}{2} - \beta.</math> (One could probably cite this as well-known, but I have ...\frac{AO}{AQ}.</math> We also have <math>\angle PAH = \angle OAQ = \frac{\pi}{2} - \beta,</math> so <math>\triangle PAH\sim\triangle OAQ</math> by SAS s
    10 KB (1,733 words) - 19:15, 14 June 2020
  • \textbf{(B) }15/\pi\qquad ...s, draw the diameter of the circle that contains <math>OD</math>). Let the two endpoints of this diameter be <math>P</math> and <math>Q,</math> where <mat
    2 KB (361 words) - 08:05, 9 April 2023
  • ...centric circles. It is <math>10</math> feet wide. The circumference of the two circles differ by about: ...in the circles' diameters is simply <math>20\pi </math> feet. Using <math>\pi \approx 3</math>, the answer is <math>\fbox{C}</math>.
    814 bytes (124 words) - 12:23, 22 April 2020
  • The diameters of two circles are <math>8</math> inches and <math>12</math> inches respectively. ...a of the smaller to the area of the larger circle is <math>\frac{16\pi}{36\pi}=\boxed{\textbf{(C) }\frac{4}{9}}</math>.
    850 bytes (134 words) - 20:49, 1 April 2017
  • ...that there exists a convex <math>1990\text{-gon}</math> with the following two properties: ...1,a_2,\cdots,a_{1990}\}=\{1,2,\cdots,1990\}</math> and <math>\theta=\frac{\pi}{995}</math>.
    3 KB (522 words) - 13:54, 30 January 2021
  • ...less than or equal to <math>r</math>, <math>e.g.,</math> <math>[6] = 6, [\pi] = 3, [-1.5] = -2.</math> Indicate on the <math>(x,y)</math>-plane the set ...in the correct seat. Prove that the table can be rotated so that at least two of the guests are simultaneously correctly seated. </div>
    3 KB (499 words) - 12:17, 11 August 2016
  • <math>\textbf{(A) }\frac{\pi}{2}\qquad \textbf{(B) }\frac{\pi}{3}\qquad
    2 KB (363 words) - 12:46, 10 May 2022
  • ...th> units of line segment <math>\overline{AB}</math> has volume <math>216 \pi</math>. What is the length <math>AB</math>? ...central axis <math>\overline{AB}</math> and radius <math>3</math> and the two hemispheres connected to each face of the cylinder (also with radius <math>
    1 KB (159 words) - 14:59, 10 June 2023
  • ...ath> units of line segment <math>\overline{AB}</math> has volume <math>216\pi</math>. What is the length <math>\textit{AB}</math>? ...of our cylinder + the volume of our two hemispheres will equal <math>216 \pi</math>):
    2 KB (320 words) - 14:24, 13 April 2024
  • ...{3}-\frac{2\sqrt{3}\pi}{9}\qquad \textbf{(E) } \frac{4}{3}-\frac{4\sqrt{3}\pi}{27}</math> ...}{\frac{3r^2 \sqrt{3}}4} = \boxed{\textbf{(E) } \frac 43 - \frac{4\sqrt 3 \pi}{27}}</cmath>
    6 KB (993 words) - 20:28, 23 February 2024
  • Tamara has three rows of two <math>6</math>-feet by <math>2</math>-feet flower beds in her garden. The b ...th>4</math> times their product. What is the sum of the reciprocals of the two numbers?
    15 KB (2,285 words) - 18:02, 28 October 2023
  • Between <math>1</math> and <math>\frac{\pi}{2}</math>, <math>f(x)</math> starts at a positive number and increases to Between <math>\frac{\pi}{2}</math> and 3, <math>f(x)</math> starts at <math>-\infty</math> and incr
    3 KB (564 words) - 14:12, 23 October 2021
  • ...e of odd multiples of <math>\pi/4</math>, i.e. <math>\pi/4,3\pi/4,5\pi/4,7\pi/4</math>. When we draw these 6 complex numbers out on the complex plane, we ...\sum\theta\equiv\pi \mod 2\pi</math>. Letting <math>z</math> be <math>e^{i\pi/4}</math>, we see that the angles we have available are <math>\{z^1,z^2,z^3
    18 KB (2,878 words) - 01:47, 16 December 2023
  • ...umbers is 4 times their product. What is the sum of the reciprocals of the two numbers? ...th> units of line segment <math>\overline{AB}</math> has volume <math>216 \pi</math>. What is the length <math>AB</math>?
    15 KB (2,418 words) - 16:58, 7 November 2022
  • ...(x)</math> and <math>\cos(x)</math> are periodic with least period <math>2\pi</math>. What is the least period of the function <math>\cos(\sin(x))</math> ...i}{2}\qquad\textbf{(B)}\ \pi\qquad\textbf{(C)}\ 2\pi \qquad\textbf{(D)}\ 4\pi \qquad\textbf{(E)}</math> It's not periodic.
    15 KB (2,343 words) - 18:26, 25 December 2020
  • ...has base <math>\frac{\sqrt{6}}{2}</math>. Adding these and multiplying by two, we get the sum of the four roots as <math>\sqrt{2} + \sqrt{6}</math>. Howe ...are <math>\cos(0), \pm\cos(\frac{\pi}{6}),</math> and <math>\pm\cos(\frac{\pi}{3})</math>. Their sum is <math>1+2(\frac{\sqrt{3}}{2}+\frac{1}{2})=1+\sqrt
    3 KB (449 words) - 01:54, 11 February 2019
  • ...he interval <math>(0, 75)</math>. Let <math>O</math> and <math>P</math> be two points on the plane with <math>OP = 200</math>. Let <math>Q</math> and <mat ...</math> by <math>75</math> box in the <math>ab</math> plane. This cuts off two isosceles right triangles from opposite corners with side lengths <math>45<
    9 KB (1,539 words) - 15:47, 17 February 2024
  • ...subtract two vectors, the geometric result is the line segment between the two endpoints of the vectors. Thus we can fill in <math>z_3 - z_1, z_2 - z_1, z ...ath>, so let’s try to prove that they are congruent. We can show this in two ways;
    13 KB (2,252 words) - 15:46, 6 January 2024
  • ...>(0,0),(5,0),(0,2\sqrt{3}),</math> and let <math>(a,0),(0,b)</math> be the two vertices of the equilateral triangle on the legs of the right triangle. The We know that <math>(-a+bi)\cdot\cos(-\frac{\pi}{3})=(-\frac{a+\sqrt{3}b}{2}+\frac{\sqrt{3}a+b}{2}i)</math>. We know that t
    22 KB (3,622 words) - 17:11, 6 January 2024
  • ...labels occur, and for each label <math>i</math>, the distance between any two points labeled <math>i</math> is at least <math>c^i</math>. ...oints on the coordinate plane, but with the minimum separation between any two points equaling <math>x</math> (as opposed to <math>1</math>).
    8 KB (1,495 words) - 12:19, 17 July 2023
  • ...ditional unit squares that must be shaded so that the resulting figure has two lines of symmetry<math>?</math> ...>\tan{(2x)} = \cos{(\tfrac{x}{2})}</math> have on the interval <math>[0, 2\pi]?</math>
    14 KB (2,073 words) - 15:15, 21 October 2021
  • ...e. What is the greatest possible value of the product of the slopes of the two lines? ...the lateral surface of a right circular cone by taping together along the two radii shown. What is the volume of the cone in cubic inches?
    16 KB (2,417 words) - 01:03, 28 April 2022
  • ...{3}\qquad\textbf{(D) }4\sqrt{3}-\frac{2\pi}{3}\qquad\textbf{(E) }4+\frac{4\pi}{3}</math> ...th> Thus, our final answer is <math>\boxed{\textbf{(B)}\ 4\sqrt{3}-\frac{4\pi}{3}}.</math>
    4 KB (711 words) - 21:00, 16 January 2024
  • ...th>. With <math>\beta</math> being a real number such that <math>0< \beta<\pi/8</math> and <math>x\neq0</math>, the value of <math>\beta</math> is: (a) <math>\frac{\pi}{9}</math>
    8 KB (1,278 words) - 09:46, 11 January 2018
  • ...a national math competition held in the United Kingdom. Solvers who score over a certain threshold in the Senior Mathematical Challenge are automatically The British Mathematical Olympiad is divided into two rounds. In the first round (BMO 1), solvers have 3.5 hours to solve 6 prob
    6 KB (1,034 words) - 10:12, 7 June 2023
  • ...eta</math> is the argument of <math>z</math> such that <math>0\leq\theta<2\pi.</math> <p> ...ath> or <math>\theta\in\biggl(0,\frac{\pi}{4}\biggr)\cup\biggl(\pi,\frac{5\pi}{4}\biggr).</math> We conclude that <math>(r,\operatorname{cis}\theta)=\lef
    10 KB (1,662 words) - 12:45, 13 September 2021
  • ...gebra, geometry, and number theory -- in a <math>6</math>-period day if no two mathematics courses can be taken in consecutive periods? (What courses the ...st subset of values of <math>y</math> within the closed interval <math>[0,\pi]</math> for which
    15 KB (2,380 words) - 18:52, 7 April 2022
  • ...math>. What is the distance between the <math>x</math>-intercepts of these two lines? \textbf{(A) }25\pi \qquad
    14 KB (2,118 words) - 15:36, 28 October 2021
  • ...The triangle inequality tells us that <math>a + b > c</math>. So, we have two inequalities: So, the area is <math>\frac{\pi}{4} - \frac12</math> which is approximately <math>\boxed{\textbf{(C)} ~0.29
    3 KB (509 words) - 22:17, 25 February 2024
  • ...eta</math> is the argument of <math>z</math> such that <math>0\leq\theta<2\pi.</math> ...{4},\frac{\pi}{2},\frac{3\pi}{4},\pi,\frac{5\pi}{4},\frac{3\pi}{2},\frac{7\pi}{4}.</math></li><p>
    11 KB (1,708 words) - 12:01, 18 March 2023
  • ...placed in the plane so that each circle is externally tangent to the other two. Points <math>P_1</math>, <math>P_2</math>, and <math>P_3</math> lie on <m ...e each <math>2\pi/3</math>. Also, the distances between the centers of any two of the <math>3</math> given circles are each <math>8</math>.
    13 KB (2,080 words) - 19:09, 21 October 2023
  • ...r of mass) of <math>\triangle ABC</math> traces out a closed curve missing two points. To the nearest positive integer, what is the area of the region bou ...n below, <math>\triangle ABC_1</math> and <math>\triangle ABC_2</math> are two shapes of <math>\triangle ABC</math> with centroids <math>G_1</math> and <m
    4 KB (647 words) - 14:19, 5 June 2023
  • ...of the two vectors of norm <math>1</math> makes an angle of <math>\le\frac\pi 2</math> with the vector of norm <math>\ge 1</math>, so their sum has norm
    1 KB (227 words) - 15:45, 29 January 2021
  • The volume <math>V = \pi R^2H</math> is to be increased by the same fixed positive amount when <math \textbf{(E)}\ \text{two real values of x} </math>
    1 KB (245 words) - 19:07, 17 May 2018
  • ...sin \theta = \sin (\theta + 2\pi)</math>. We can use these facts to create two types of solutions: <cmath>\sin \theta = \sin ((2m + 1)\pi - \theta)</cmath>
    7 KB (1,211 words) - 00:23, 20 January 2024
  • ...s in the case as <math>x, y</math> respectively. Then in this case we have two subcases; ...ut the remaining cases, we can use this symmetry. The total number of ways over all the cases is <math>\sum_{k=0}^{6} W_{3k} = 2 \cdot (1+56+336)+580 = 136
    26 KB (4,044 words) - 13:58, 24 January 2024
  • The wheel shown below consists of two circles and five spokes, with a label at each point where a spoke meets a c ...e moves be <math>C</math> and clockwise moves be <math>C'</math>. The only two feasible solutions for <math>(C,C')</math> are <math>(4,9)</math> and <math
    11 KB (1,934 words) - 12:18, 29 March 2024
  • ...hat <math>y=10e^{\frac{5\pi i}{4}}\sqrt{2}</math> and <math>z=-3e^{\frac{7\pi i}{4}}\sqrt{2}</math>. Then plug <math>z</math> into <math>zx</math>, you c ...>\frac{\pi}{4}</math>, and the argument of <math>z</math> is <math>-\frac{\pi}{4}</math>. We need to convert the polar form to a standard form. Simple tr
    11 KB (2,077 words) - 20:15, 12 January 2024
  • ...ight)^2\end{align}</cmath>Eliminating <math>\cos\theta</math> in the above two equations and solving for <math>\cos\phi</math> we get<cmath>\cos\phi = \fr ...is clear that <cmath>x^2+y^2-100=2 xy \cdot \cos{APB}=-(2 xy \cdot \cos{(\pi-CPB)})=-(x^2+y^2-196) </cmath> or <cmath>\begin{align}x^2+y^2=148\end{align
    18 KB (2,912 words) - 13:12, 24 January 2024
  • ...wormhole is 2400 units. Your wormhole distance allotment is <math>\textit{two}</math>."' ...<math>3840</math> units. Your wormhole distance allotment is <math>\textit{two}</math>."
    7 KB (1,092 words) - 19:05, 17 December 2021
  • ...se of two [[constant|constants]] - <math>90^{\circ}</math> or <math>\frac{\pi}{2}</math> and <math>1.5</math> makes the [[equation|formula]] more legible ...ee (geometry)|degrees]] but if Bibhorr sthiron is in the form <math>\frac{\pi}{2}</math> then Bibhorr [[angle]] results in [[radian|radians]].
    2 KB (245 words) - 06:45, 9 September 2018
  • ...he edges of the room and to fill in the rest of the floor with two-foot by two-foot square tiles. How many tiles will she use? Abby, Bridget, and four of their classmates will be seated in two rows of three for a group picture, as shown.
    14 KB (2,191 words) - 03:19, 2 April 2024
  • ...f each of the two smaller circles is a radius of the larger circle. If the two smaller circles have a combined area of <math>1</math> square unit, then wh ...extbf{(C) } \frac{1}{2} \qquad \textbf{(D) } 1 \qquad \textbf{(E) } \frac{\pi}{2}</math>
    3 KB (454 words) - 10:15, 10 April 2024
  • ...at <math>(2,2)</math>. What is the area of the triangle enclosed by these two lines and the line <math>x+y=10 ?</math> ...s in a plane, there are exactly <math>N</math> distinct points that lie on two or more of the lines. What is the sum of all possible values of <math>N</ma
    13 KB (2,024 words) - 16:07, 22 April 2024
  • ...ath>PQ</math> intersects the circumcircle of <math>\triangle ABC</math> in two distinct points, <math>X</math> and <math>Y</math>. Suppose <math>XP=10</ma ...h>, and its angles are <math>\pi-2A</math>,<math>\pi-2B</math>, and <math>\pi-2C</math>. We thus have the three sides of the orthic triangle now.
    7 KB (1,115 words) - 03:11, 7 January 2024
  • ...nteger. Prove that <math>f(x)</math> cannot be expressed as the product of two nonconstant polynomials with integer coefficients. ...ide acute triangle <math>ABC</math> such that <math>\angle ADB=\angle ACB+\pi/2</math> and <math>AC\cdot BD=AD\cdot BC</math>.
    2 KB (452 words) - 17:59, 24 March 2019
  • ...lateral surface area of a right circular cone by taping together along the two radii shown. What is the volume of the cone in cubic inches? ...f{(C)}\ 3 \pi \sqrt7 \qquad\textbf{(D)}\ 6\pi \sqrt3 \qquad\textbf{(E)}\ 6\pi \sqrt7</math>
    16 KB (2,497 words) - 21:14, 12 November 2023
  • ...points are on the same line), then there is a line which contains exactly two of the Let <math>A</math> and <math>B</math> be two points in the plane. Describe the set <math>S</math> of all points in the p
    4 KB (604 words) - 03:48, 12 January 2019
  • Consider the two functions <math>f(x) = x^2+2bx+1</math> and <math>g(x) = 2a(x+b)</math>, wh \textbf{(B)} \ \pi \qquad
    2 KB (267 words) - 00:59, 20 February 2019
  • surface s=surface(f,(0,0),(pi,2pi),70,Spline); .../math>, so <math>\triangle{BIE} \sim \triangle{BDA}</math> since they have two equal angles.
    7 KB (1,000 words) - 15:03, 23 October 2021
  • ...at <math>(2,2)</math>. What is the area of the triangle enclosed by these two lines and the line <math>x+y=10 ?</math> ...other solutions, solve the systems of equations to see that the triangle's two other vertices are at <math>(4, 6)</math> and <math>(6, 4)</math>. Then app
    7 KB (1,079 words) - 22:24, 10 November 2023
  • Alicia had two containers. The first was <math>\tfrac{5}{6}</math> full of water and the s Two jars each contain the same number of marbles, and every marble is either bl
    15 KB (2,458 words) - 23:52, 12 November 2023
  • <cmath>\frac{a}{b}\cdot\pi-\sqrt{c}+d,</cmath> The area of <math>A</math> is <math>\frac{1}{2} \pi \cdot 2^2 = 2\pi</math>.
    6 KB (984 words) - 23:52, 11 November 2023
  • <math>\textbf{(A) }\frac{83\pi}{8}\qquad\textbf{(B) }\frac{21\pi}{2}\qquad\textbf{(C) } \frac{85\pi}{8}\qquad\textbf{(D) }\frac{43\pi}{4}\qquad\textbf{(E) }\frac{87\pi}{8}</math>
    6 KB (967 words) - 10:25, 20 December 2023
  • Alicia had two containers. The first was <math>\tfrac{5}{6}</math> full of water and the s ...gle <math>ABC</math> with leg length <math>1</math>, as shown, so that the two triangles have equal perimeters. What is <math>\sin(2\angle BAD)</math>?
    16 KB (2,477 words) - 15:41, 9 September 2023
  • ...< \theta < \pi</math>. By symmetry, the interval <math>\pi \leq \theta < 2\pi</math> will also give <math>2</math> solutions. The answer is thus <math>2 ...rm an equilateral triangle, their difference in angle must be <math>\frac{\pi}{3}</math>, so
    6 KB (987 words) - 19:19, 12 November 2022
  • ...c{3}{2}\sqrt3\qquad\textbf{(D) } \frac{1}{2}\pi\sqrt3 \qquad\textbf{(E) } \pi</math> Notice that <math>\omega=e^{\frac{2i\pi}{3}}</math>, which is one of the cube roots of unity. We wish to find the s
    4 KB (729 words) - 21:23, 15 November 2022
  • ...ex numbers you subtract the angle of one from the other. Therefore, if the two complex numbers are perpendicular, the difference between their arguments w According to the question <math>\arg{Z_1}=\frac{\pi}{2}</math>.
    8 KB (1,534 words) - 22:17, 28 December 2023
  • π (the famous number pi that turns up in many interesting areas) And here is the miracle ... the two groups are quite neatly the Taylor Series for cos and sin
    3 KB (543 words) - 15:24, 13 June 2019
  • ...ve number would have an additive inverse. ... The fact that the product of two negatives is positive is therefore related to the fact that the inverse of A fraction a number that can be expressed as two numbers divided. For example, five divided by four is <math>\frac{5}{4}</ma
    35 KB (5,882 words) - 18:08, 28 June 2021
  • (Hint: <math> \sum_{n=1}^{\infty}\frac{1}{n^2}=\frac{\pi^2}{6} </math>) <cmath>\prod_{primes} (1-p^{-2}) = \frac{1}{\xi(2)} = \frac{6}{\pi^2}</cmath>.
    3 KB (445 words) - 18:37, 14 January 2020
  • ...math>\frac{\pi \cdot r^2}{2} = \frac{\pi \cdot (\pi)^2}{2} = \boxed{\frac{\pi^3}{2}}</cmath>
    544 bytes (99 words) - 18:36, 13 January 2020
  • The set of complex numbers is a set of all numbers ever existed, from pi to 1. The set of all real numbers can be classified into two major sub sets, the set of irrational and rational numbers.
    2 KB (270 words) - 16:28, 13 June 2022
  • ...or a boating trip. How many ways are there to split up the six boys if the two groups are indistinguishable? ...\sqrt{3})^{2019} \qquad \textbf{(C) }(3+\sqrt{2})^2 \qquad \textbf{(D) }(2\pi)^2 \qquad \textbf{(E) }(3+\sqrt{2})(3-\sqrt{2}) \qquad</math>
    13 KB (2,059 words) - 02:59, 21 January 2021
  • ...h>a = -b</math> or <math>a^2 + b^2 + a - b = 0</math>. We can divide into two cases. ...ath> to get <math>\tan{x} = -1</math>. Thus, <math>x = \frac{3 \pi}{4} + \pi n</math>, where <math>n</math> is an integer.
    2 KB (305 words) - 06:07, 23 February 2023
  • 4. The Infinity Numeral, PI, is the second deity of the Almighty Gmaas's heaven. ...te dreadful. EDIT: He now eats gnats again. He thinks that they taste like pi(e). EDIT: He ate 3,141,592,653,589,793,238,462,643,383 so far.
    85 KB (13,954 words) - 17:25, 22 March 2024
  • How many times per day do at least two of the three hands on a clock coincide? ...eed <math>v_b</math>. What is the distance of closest approach between the two?
    4 KB (618 words) - 13:33, 21 January 2020
  • ...each couple, and it's unique for every couple. In every 3 persons, exactly two of them is not communicating to each other. Determine the maximum number of <cmath> \frac{\pi (a^{2}+b^{2}+c^{2})(b+c-a)(c+a-b)(a+b-c)}{(a+b+c)^{3}}.</cmath>
    3 KB (478 words) - 14:09, 23 June 2021
  • In the isosceles right triangle, the two legs are congruent. We can therefore construct an isosceles right triangle ...</math> and equal to the original line segment. The difference between the two vectors is <math><2,8></math>, which is the slope <math>4</math>, and that
    7 KB (1,145 words) - 20:27, 5 November 2023
  • <cmath>\sin^2{(\pi x)} + \sin^2{(\pi y)} > 1</cmath> <cmath>\sin^{2}{(\pi x)}+\sin^{2}{(\pi y)}>1</cmath>
    8 KB (1,412 words) - 06:17, 30 December 2023
  • ...lateral surface area of a right circular cone by taping together along the two radii shown. What is the volume of the cone in cubic inches? ...f{(C)}\ 3 \pi \sqrt7 \qquad\textbf{(D)}\ 6\pi \sqrt3 \qquad\textbf{(E)}\ 6\pi \sqrt7</math>
    3 KB (493 words) - 16:14, 11 September 2023
  • ...quad \textbf{(D) } 3\sqrt3 - \pi \qquad \textbf{(E) } \frac{9\sqrt3}{2} - \pi</math> ...ing vertex, and let point <math>C</math> be the second intersection of the two semicircles that pass through point <math>A</math>. Then, <math>BC = 1</mat
    17 KB (2,392 words) - 12:36, 24 December 2023
  • ...chool has <math>3</math> times as many students as Lara's high school. The two high schools have a total of <math>2600</math> students. How many students ...r is erased, the other number is obtained. What is the difference of these two numbers?
    15 KB (2,302 words) - 23:41, 14 April 2024
  • ...rt{3}}{2}</math> = <math>\cos(120^\circ) + i \cdot \sin(120^\circ) = e^{ 2\pi i / 3}</math>. ...p)</math> which are <math>(1,2,2)</math> and <math>(3,1,1)</math> yielding two possible polynomials. The answer is thus <math>\boxed{\textbf{(C) } 2}</mat
    4 KB (726 words) - 16:55, 11 September 2023
  • ...th> to be equal to <math>e^{i\frac{2\pi}{3}}</math> and <math>e^{-i\frac{2\pi}{3}}</math>, meaning that all three are equally spaced along the unit circl ...e condition in the problem holds for some <math>n=k</math>. We can now add two points <math>z_{k+1}</math> and <math>z_{k+2}</math> anywhere on the unit c
    2 KB (306 words) - 17:45, 28 January 2024
  • A = (0, tan(3 * pi / 7)); ...{BAC}</math> to <math>x</math>, we can find all other angles through these two properties:
    6 KB (968 words) - 15:01, 24 January 2024
  • Dr.Jeck is arranging books on a bookshelf. How many ways are there to arrange two identical math books, three identical science books and four identical Engl ...4</math>, a quarter circle arc of radius <math>\sqrt{10}</math> intersects two sides that divided the side length into ratio <math>3:1</math>, as shown in
    11 KB (1,745 words) - 16:44, 6 October 2021
  • ...o opposite corners of the index card and measures the distance between the two closest vertices of these squares to be <math>4\sqrt{2}</math> centimeters, ...</math> What is the sum of all real numbers <math>m</math> for which these two expressions have the same value?
    18 KB (2,662 words) - 02:08, 9 March 2024
  • A plane flies at a speed of <math>590</math> miles/hour. How many miles in two hours ...\pi}{13}</math>, <math>\cos \frac{5\pi}{13}</math>, and <math>\cos \frac{7\pi}{13}</math>. What is the least possible sum of the coefficients of <math>P(
    8 KB (1,223 words) - 15:02, 27 November 2022
  • ...[[Mathcounts Week Countdown Round]]. Most of the times of the activity are pi time (3:14 PM). ...not saving for the mock sprint and target competitions. There are at least two incidents of incorrect answers being given points in the "World Largest Cou
    1 KB (207 words) - 17:51, 22 May 2020
  • ...'true' powers and the primes. Then this would give that <math>\frac{p(n)+\pi(n)}{n} \geq \frac{1}{k}</math> for all <math>n</math> in contrary to the ab Edit: to elementarize the <math>\lim_{n \to \infty} \frac{\pi(n)}{n} = 0</math> part:
    2 KB (390 words) - 21:58, 4 July 2021
  • Let the circles <math>k_1</math> and <math>k_2</math> intersect at two points <math>A</math> and <math>B</math>, and let <math>t</math> be a commo ...>, <math>\angle PMB \cong \boxed {\angle NMB \cong 45^{\circ} \cong \frac{\pi}{4}}</math>.
    2 KB (392 words) - 15:47, 11 January 2021
  • ...a</math> is <math>1</math>, is achieved at <math>\theta = \frac{\pi}{2}+2k\pi</math> for some integer <math>k</math>. ...that <math>mx = \frac{\pi}{2}+2a\pi</math> and <math>nx = \frac{\pi}{2}+2b\pi</math>, for integers <math>a, b</math>.
    9 KB (1,523 words) - 09:12, 3 December 2023
  • ...r is erased, the other number is obtained. What is the difference of these two numbers? Two right circular cones with vertices facing down as shown in the figure below
    15 KB (2,383 words) - 09:49, 25 June 2023
  • ...eta</math> is the argument of <math>z</math> such that <math>0\leq\theta<2\pi.</math> \textbf{(A)} & & 2 & & & \pi & & & &32\cos{(5\pi)}&=&32\cos\pi&=&32(-1)& & \\ [2ex]
    6 KB (872 words) - 17:36, 4 December 2021
  • ...the set of all positive rational numbers <math>r</math> such that when the two numbers <math>r</math> and <math>55r</math> are written as fractions in low ...region formed by three unit squares joined at their sides, as shown below. Two points <math>A</math> and <math>B</math> are chosen independently and unifo
    8 KB (1,370 words) - 21:34, 28 January 2024
  • How many integer values of <math>x</math> satisfy <math>|x| < 3\pi</math>? ...he cousins' ages multiplied together give <math>24</math>, while the other two multiply to <math>30</math>. What is the sum of the ages of Jonie's four co
    17 KB (2,418 words) - 12:52, 5 November 2023
  • ...xtbf{(C)}\ 12\sqrt{3}+14\pi\\ \textbf{(D)}\ 12+15\pi\qquad\textbf{(E)}\ 24\pi </math> We can separate the total length of the wire into two sections: the straight and the curve.
    2 KB (360 words) - 22:07, 16 August 2020
  • ...th> and <math>CY=20</math>. We can compute the area of <math>ABC</math> in two ways: Equating the two expressions for <math>[ABC]</math> and solving for <math>r</math> yields <m
    16 KB (2,517 words) - 20:22, 31 January 2024
  • ...passes through <math>P</math> and divides <math>\triangle ABC</math> into two polygons of equal perimeter. Let <math>\triangle ABC</math> be a triangle w ...th>r^2+r+1=(r-\omega)(r-{\omega}^2)</math>, where <math>\omega=e^{i\frac{2\pi}{3}}</math>. Thus,
    16 KB (2,730 words) - 02:56, 4 January 2023
  • We can symmetrically apply this to the two other equations/triangles. Taking the inverse sine (<math>0\leq\theta\frac{\pi}{2}</math>) of each equation yields a simple system:
    15 KB (2,208 words) - 01:25, 1 February 2024
  • ...<math>75\pi+50</math>. The greatest integer less than or equal to <math>75\pi+50</math> is <math>\boxed{285}</math>.
    887 bytes (154 words) - 17:37, 4 October 2020
  • ...constant <math>k</math> will the polynomial <math>x^{2}+kx+36</math> have two distinct integer roots? How many of these sets contain exactly two multiples of <math>7</math>?
    15 KB (2,233 words) - 13:02, 10 November 2023
  • ...has risen by <math>3</math> degrees, at which time the temperatures in the two cities differ by <math>2</math> degrees. What is the product of all possibl ...s <math>15</math>. How many distinct integers can be written as the sum of two, not necessarily different, special fractions?
    16 KB (2,450 words) - 00:13, 12 November 2023
  • Mr. Lopez has a choice of two routes to get to work. Route A is <math>6</math> miles long, and his averag ...sum of its nonzero tens digit and the square of its units digit. How many two-digit positive integers are cuddly?
    14 KB (2,162 words) - 21:33, 2 November 2023
  • ...> We construct three segments <math>PA, PB,PC</math>, perpendicular two by two<math>,</math> with the vertexes <math>A, B, C</math> on the sphere<math>.</ ...| PB \right|^{2} - 2 \left| OP \right| \left| PB \right| cos \left( \frac{\pi}{2}-\theta \right)</math>
    6 KB (1,039 words) - 13:00, 13 March 2024
  • ...dicular bisector of <math>AP,BP</math> meet at point <math>O</math>, those two lines meet at <math>AD,BC</math> at <math>N,M</math> respectively. <cmath>\angle APE = \alpha, \angle PEA = \pi - 2\alpha, \angle ABP = 2\alpha \implies</cmath>
    3 KB (464 words) - 08:29, 28 September 2023
  • Two right circular cones with vertices facing down as shown in the figure below \textbf{Narrow Cone} & 3 & h_1 & & \frac13\pi(3)^2h_1=3\pi h_1 & \\ [2ex]
    9 KB (1,503 words) - 15:09, 1 August 2023
  • ...ft( \frac{\pi}2 \sin x\right)</math> have in the closed interval <math>[0,\pi]</math>? ...<math>\frac{\pi}2 \cos x</math> are both <math>\left[-\frac{\pi}2, \frac{\pi}2 \right],</math> which is included in the range of <math>\arcsin,</math> s
    7 KB (1,110 words) - 20:10, 5 November 2022
  • [*] also suppose two consecutive lines are one unit apart to each other . ...that <math>l_k \equiv \ Im ({P_k})=y \cos \frac{2\pi k}{n} +x \sin \frac{2\pi k}{n}</math> .
    3 KB (607 words) - 03:12, 22 September 2023
  • How many integer values of <math>x</math> satisfy <math>|x|<3\pi?</math> Ms. Blackwell gives an exam to two classes. The mean of the scores of the students in the morning class is <ma
    15 KB (2,302 words) - 12:31, 27 October 2023
  • A worker cuts a piece of wire into two pieces. The two pieces, <math>A</math> and <math>B</math>, enclose an equilateral triangle ...s into <math>3</math> different boxes such that each box contains at least two balls, and that no box can contain <math>7</math> or more balls. Find the n
    15 KB (2,366 words) - 17:45, 19 September 2021
  • ...they just had a fight. Find the number of ways the coach can put them into two such groups. ...f{(C)} ~\frac{(2-\sqrt{2})\pi}{8-2\pi} \qquad\textbf{(D)} ~\frac{\pi}{32-8\pi}\qquad</math>
    12 KB (1,915 words) - 17:38, 29 April 2021
  • ...of <math>7\pi</math> and the total area covered by the circles is <math>25\pi</math>. What is the value of <math>r</math>? There are <math>8</math> scoops of ice cream, two of each flavor: vanilla, strawberry, cherry, and mint. If scoops of the sam
    5 KB (776 words) - 09:35, 8 August 2023
  • ...of <math>7\pi</math> and the total area covered by the circles is <math>25\pi</math>. What is the value of <math>r</math>? ...ac{25\pi+7\pi}{2}=16\pi.</cmath> The radius of a circle with area <math>16\pi</math> is <math>\boxed{\textbf{(C) } 4}</math>.
    879 bytes (135 words) - 21:48, 3 January 2021
  • has more than one solution in the interval <math>(0, \pi)</math>. The set of all such <math>a</math> that can be written ...erings of the cards will the <math>13</math> cards be picked up in exactly two passes?
    15 KB (2,250 words) - 00:32, 9 March 2024
  • ...\qquad \textbf{(C) }64\pi \qquad \textbf{(D) }65\pi \qquad \textbf{(E) }84\pi</math> ...ius <math>1,</math> in which we do not need to subtract more. That is, the two smallest circles are on the same side of <math>\ell,</math> but can be on e
    2 KB (363 words) - 18:22, 16 August 2022
  • How many integer values of <math>x</math> satisfy <math>|x|<3\pi</math>? Since <math>3\pi\approx9.42</math>, we multiply <math>9</math> by <math>2</math> for the int
    4 KB (581 words) - 23:48, 18 July 2023
  • ...the volume of the water is <math>\frac{1}{3}\cdot18\cdot144\pi = 6\cdot144\pi</math>. ...> so the volume of the water in the cylinder would be <math>24\cdot24\cdot\pi\cdot h</math>.
    2 KB (405 words) - 23:51, 18 July 2023
  • z &= e^{i 0}, e^{i \frac{2\pi}{3}}, e^{i\left(-\frac{2\pi}{3}\right)}. Obviously the right two solutions are the roots of <math>z^2 + z + 1 = 0.</math>
    8 KB (1,315 words) - 11:43, 24 October 2023
  • How many values of <math>\theta</math> in the interval <math>0<\theta\le 2\pi</math> satisfy <cmath>1-3\sin\theta+5\cos3\theta = 0?</cmath> We can graph two functions in this case: <math>y=5\cos{3x}</math> and <math>y=3\sin{x} -1 </
    2 KB (250 words) - 18:57, 19 September 2023
  • from the point to the origin is at most two units? \textbf{(D) }\frac{\pi}{16}\qquad
    519 bytes (83 words) - 19:07, 15 March 2024
  • <center><math>C_m=\mathcal{C}(t;z_0+r_me^{it},0,2\pi)</math></center> <center><math>2\pi f(w)=\oint_{C_2}\frac{f(z)}{z-w}\,dz-\oint_{C_1}\frac{f(z)}{z-w}\,dz</math>
    8 KB (1,471 words) - 22:02, 12 April 2022
  • ...ss than or equal to <math>n</math>. Suppose <math>\pi(a)^{\pi(b)}=\pi(b)^{\pi(a)}=c</math>. For some fixed <math>c</math> what is the maximum possible nu ...the two are related by the formula <math>F=\tfrac{9}{5}C+32</math>. When a two-digit integer degree temperature <math>n</math> in Celcius is converted to
    4 KB (651 words) - 20:18, 6 March 2021
  • A worker cuts a piece of wire into two pieces. The two pieces, <math>A</math> and <math>B</math>, enclose an equilateral triangle ...s into <math>3</math> different boxes such that each box contains at least two balls, and that no box can contain <math>7</math> or more balls. Find the n
    14 KB (2,226 words) - 23:39, 12 September 2021
  • ...> and line segments <math>BO</math> and <math>GH</math>) has area <math>25\pi</math>. Furthermore, another quarter circle <math>DOF</math> formed by arc ...extbf{(E)} ~\frac{50\pi^2+700\pi\sqrt{2}+3001\pi-70\sqrt{2}+60}{2\pi^2+240\pi+6920}\qquad</math>
    3 KB (520 words) - 14:04, 26 April 2021
  • ...i} \qquad\textbf{(D)} ~\frac{13}{28+6\pi} \qquad\textbf{(E)} ~\frac{13}{30\pi}\qquad</math> ...atio would be <math>\frac{26}{56+12\pi}=\boxed{\textbf{(E)} \frac{13}{28+6\pi}}</math>.
    3 KB (599 words) - 14:05, 26 April 2021
  • <math>\textbf{(A)} ~4 \qquad\textbf{(B)} ~\frac{12}{\pi} \qquad\textbf{(C)} ~\frac{2+3\sqrt{3}}{2} \qquad\textbf{(D)} ~\frac{4+\sqr ...d intersection of <math>\frac{6}{\pi}\cdot2=\boxed{\textbf{(B)} \frac{12}{\pi}}</math>.
    1 KB (228 words) - 14:06, 26 April 2021
  • <math>\textbf{(A)}</math> Two overlapping circles with each area <math>2\pi</math>. <math>\textbf{(B)}</math> Four not overlapping circles with each area <math>4\pi</math>.
    1 KB (223 words) - 13:54, 26 April 2021
  • Given that the two roots of polynomial <math>x^{2}-ax+\frac{1}{2}</math> are <math>\sec(n)</ma ...} \qquad\textbf{(D)} ~\frac{\pi+2\sqrt{3}-3}{4} \qquad\textbf{(E)} ~\frac{\pi+3-2\sqrt{2}}{4}</math>
    13 KB (2,097 words) - 17:38, 29 April 2021
  • ...LL</math> civilization, each number digits of a number will be replaced by two times of the digit. For example: <math>1234</math> in <math>MICWELL</math> Statement 4: The length of a diagonal is the product of two adjacent sides.
    11 KB (1,691 words) - 18:56, 25 April 2022
  • <math>\triangle AMB' \sim \triangle MDC'</math> by two angles. ...</math>. Then <math>\angle{AMD}=\pi-(a+b), \angle{ABM}=\pi-c, \angle{DCM}=\pi-c</math>, implying that <math>\angle{BMA}+\angle{CMD}=a+b</math>, implying
    14 KB (2,254 words) - 18:26, 8 February 2024
  • Two externally tangent circles <math>\omega_1</math> and <math>\omega_2</math> <math>\angle B'O_2 D > \frac{\pi}{2},</math> hence <math>\cos \angle B'O_2 D = \cos 2\alpha = -\frac{3}{5}.<
    14 KB (2,217 words) - 00:28, 29 June 2023
  • If the cross sections of two 2D objects at each height have the same length, the areas of the 2D objects If the cross sections of two 3D objects at each height have the same area, the volumes of the 3D objects
    1 KB (167 words) - 22:01, 27 August 2021
  • ...constant <math>k</math> will the polynomial <math>x^{2}+kx+36</math> have two distinct integer roots? How many of these sets contain exactly two multiples of <math>7</math>?
    15 KB (2,224 words) - 13:10, 20 February 2024
  • What two-digit even number has digits that sum to <math>17</math>? How many positive two-digit numbers exist such that the product of its digits is not zero?
    7 KB (1,100 words) - 18:40, 11 July 2021
  • ...h>. Connect <math>IM,IT,IT,IZ,IX,IN,IJ,IF</math> separately, we can create two pairs of congruent triangles. In <math>\triangle{XIF},\triangle{YIM}</math> <cmath>\overset{\Large\frown} {XY}= 2\angle XAY = 2\pi - 4\alpha,</cmath>
    7 KB (1,196 words) - 10:30, 18 June 2023
  • '''then <math>y = (2n-1)\pi^c + x </math> or <math>y = (2n-1)180^\circ - x </math>''' ...nstant in the theorem <math>\pi^c </math>/<math>180^\circ </math>. <math>\pi^c </math> is used if the angle <math>x </math> is in [https://en.wikipedi
    2 KB (374 words) - 03:28, 29 July 2021
  • ...\textbf{(C)} \: 18\pi \qquad\textbf{(D)} \: 24\pi \qquad\textbf{(E)} \: 27\pi</math> The area is therefore <math> \pi r^2 = \boxed{\textbf{(B)}\ 12\pi}</math>.
    5 KB (765 words) - 22:33, 10 July 2023
  • ...h>2A</math>. The value of <math>s</math> can be written as <math>a+\frac{b\pi}{c}</math>, where <math>a,b</math>, and <math>c</math> are positive integer real s = 5 + pi/4;
    3 KB (475 words) - 17:45, 30 October 2023
  • determined by a side of the hexagon is reflected over that side. What is the area of the region ...\qquad(\textbf{D}) \: \pi - \frac{\sqrt{3}}{2}\qquad(\textbf{E}) \: \frac{\pi + \sqrt{3}}{2}</math>
    6 KB (1,021 words) - 19:40, 1 November 2023
  • Mr. Lopez has a choice of two routes to get to work. Route A is <math>6</math> miles long, and his averag Consider two concentric circles of radius <math>17</math> and <math>19.</math> The large
    15 KB (2,452 words) - 19:37, 7 June 2023
  • ...has risen by <math>3</math> degrees, at which time the temperatures in the two cities differ by <math>2</math> degrees. What is the product of all possibl ...s <math>15</math>. How many distinct integers can be written as the sum of two, not necessarily different, special fractions?
    14 KB (2,191 words) - 19:57, 12 November 2023
  • ...c}</math>. If the volume the square sweeps out can be expressed as <math>m\pi</math>, find <math>m</math>. The number of ways to arrange the characters in "delicious greenbeans" into two separate strings of letters can be expressed as <math>a\cdot b!,</math> whe
    4 KB (707 words) - 12:38, 6 June 2022
  • ==Proof of equivalence of the two definitions== ...gives <math>2mk\pi \pm mny</math>; taking the cosine gives <math>\cos (2mk\pi \pm mny) = \cos mny = \cos ( mn (\arccos x)) = T_{mn}(x)</math>.
    10 KB (1,919 words) - 15:24, 26 June 2023
  • ==Proof of equivalence of the two definitions== ...</math> and the fact that the range of <math>\arccos x</math> is <math>[0,\pi]</math>, on which <math>\sin y</math> is nonnegative.
    2 KB (392 words) - 22:12, 11 March 2022
  • ...h> over the perpendicular bisector of <math>\overline{BC}</math>, creating two new isosceles trapezoids <math>DAPP^{\prime}</math> and <math>CBPP^{\prime} ...}\cdot AD=15</math> and <math>PP^{\prime}\cdot BC=5</math>; dividing these two equations and taking the reciprocal yields <math>\frac{BC}{AD}=\boxed{\text
    12 KB (1,806 words) - 12:52, 26 March 2024
  • ...th> be a real number, and let <math>z_1</math> and <math>z_2</math> be the two complex numbers satisfying the equation ...ce between the two lines, is the difference between the real values of the two bases <math>\implies h= a-\frac{a}{10}=\frac{9a}{10}</math>.
    7 KB (1,164 words) - 11:20, 1 January 2024
  • ...but exterior to the quadrilateral can be written in the form <math>\frac{a\pi-b}{c},</math> where <math>a,b,</math> and <math>c</math> are positive integ ...2\cdot AD\cdot DC = \frac{625\pi}{4}-\frac{168}{2}-\frac{300}{2}=\frac{625\pi-936}{4}.</cmath>
    5 KB (768 words) - 21:00, 21 February 2024
  • has more than one solution in the interval <math>(0, \pi)</math>. The set of all such <math>a</math> that can be written Since <math>\sin{x} \ne 0</math> as it is on the open interval <math>(0, \pi)</math>, we can divide out <math>\sin{x}</math> from both sides, leaving us
    6 KB (1,001 words) - 21:27, 17 March 2024
  • //Comment two lines below to remove red edges ...tersection by symmetry. Therefore, the triangle that is wedged between the two hexagons has the same angle as the square at the bottom wedged between the
    13 KB (2,080 words) - 11:27, 25 October 2023
  • ...o semicircles will be formed with a radius of 1 for a total area of <math>\pi \approx 3</math>. Therefore, the total area is <math>5(2) + \pi \approx 10 + 3 = \boxed{\textbf{(A) } 13}</math>.
    2 KB (344 words) - 15:41, 25 October 2023
  • ...\pi\qquad\textbf{(C)}~96\pi\qquad\textbf{(D)}~102\pi\qquad\textbf{(E)}~136\pi\qquad</math> There are four cases with two circles belonging to each:
    7 KB (1,202 words) - 01:15, 10 June 2023
  • -1 + i\sqrt{3} &= 2e^{\frac{2\pi i}{3}}, \\ -1 - i\sqrt{3} &= 2e^{-\frac{2\pi i}{3}} = 2e^{\frac{4\pi i}{3}}.
    5 KB (866 words) - 22:17, 27 October 2023
  • (ii) no two points in <math>P</math> lie on a line through the origin. ...real number <math>x</math>. Thus <math>\lfloor\sqrt{2}\rfloor = 1, \lfloor\pi\rfloor =\lfloor 22/7 \rfloor = 3, \lfloor 42\rfloor = 42,</math> and <math>
    3 KB (492 words) - 14:07, 24 December 2022
  • ...) of a <math>3 \times 3</math> grid of squares, but you are not told which two squares are covered. Your goal is to find at least one square that is cover ...ath>-axis at the origin. What is the slope of the line passing through the two points at which these circles intersect?
    13 KB (2,107 words) - 22:19, 20 April 2024
  • ...he total area of the radius <math>3</math> circle is simply just <math>9* \pi</math> when using our area of a circle formula. ...ll get <math>3 * \frac{1}{4} \pi + 4 \pi -\pi - \pi = \frac{3}{4} \pi + 2 \pi = \frac{11}{4}</math>.
    3 KB (452 words) - 14:33, 21 January 2024
  • ...maximum possible volume of this cylinder in the form of <math>\frac{a}{b}\pi</math>? <math>V=\pi{r^2}h</math>
    4 KB (697 words) - 17:07, 24 March 2023

View (previous 500 | next 500) (20 | 50 | 100 | 250 | 500)